Vous êtes sur la page 1sur 86

GMAT

FACULTY
MANUAL
(Mathematic
s)

Introduction to the Syllabus Quantitative Reasoning Measure


THE SKILLS, CONCEPTS AND ABILITIES ARE TESTED IN THE FOUR CONTENT AREAS BELOW:
ARITHMETIC TOPICS INCLUDES
1. properties and types of integers
2. Test of divisibility
3. Concept of factorization
4. Concept of prime numbers
5. Concept of remainders
6. Concept of odd and even integers
7. Arithmetic operations
8. Concept of exponents and radicals
9. Concept of estimation
10.Concept of percent
11.Concept of ratio, Proportion and variations
12.Concept of rate
13.Concept of absolute value
14.The number line
15.Concept of decimal representation
16.Concept of sequences of numbers
ALGEBRA TOPICS INCLUDES
1. Operations with exponents
2. Factoring and simplifying algebraic expressions
3. Concept of relations and functions
4. Equations and solving linear and quadratic equations and solving simultaneous
equations
5. Inequalities and solving linear and quadratic inequalities
6. Setting up equations to solve word problems
7. Graphs of functions, equations and inequalities.
GEOMETRY TOPICS INCLUDES
1. Lines angles and polygons parallel and perpendicular lines
2. Circles
3. Triangles including isosceles, equilateral and 30-60-90 triangles quadrilaterals,
other polygons, congruent and similar figures, the Pythagorean theorem and angle
measurement in degrees.
4. Quadrilaterals
5. Solid Geometry three-dimensional figures, area, perimeter, volume, the ability to
construct proofs is not tested.
6. Coordinate geometry, including slopes and intercepts of lines
COMBINATORICS TOPICS INCLUDES
1. Basic descriptive statistics
2. Mean
3. Median
4. Mode
5. Range
6. Standard deviation
7. Elementary probability
8. Probabilities of compound events

9. Probabilities of independent events


10.Random variables
11.Probability distributions
12.Normal distributions
13.Counting methods
14.Combinations
15.Permutations and Venn diagrams.

SESSION-1 (MODULE-1-ARITHMETIC)
Topics to be covered
Number Tree
Primes
Decimals, Divisibility Rules
HCF/LCM
1. Properties and types of integers
(i)
Natural Number (counting numbers)
(ii)
Whole Number (counting numbers including zero)
(iii)
Integers
(i)
Positive Integers
(ii)
Negative Integers
(iii)
Neither Positive non Negative Integer (Zero)
(iv)
Properties of Integer zero such as multiplication, division, addition and
exponent with zeros

(v)
(vi)
(vii)
(viii)
(ix)
(x)
(xi)
(xii)

0
, , , , 00 , 0 ,1
0

Discuss not defined forms such as


and explain
why these are not defined but do not go for proof.
Explain what is the meaning of not defined
Explain what is infinite
Definition of positive infinite
Definition of negative infinite
Explain why infinite is not defined
What is the difference b/w infinite and not defined
Do you think all infinites are not defined or all not defined or infinite

2. Rational Numbers (define with examples)


3. Irrational Number (define with examples)
(i)
(ii)
(iii)
(iv)

Explain why is not a rational number


What is basic difference b/w rational and irrational numbers?
What is the significance of rational and irrational numbers?
Explain how decimal and not terminating recurring decimal numbers are
rational numbers.
(v)
Explain why non terminating and non recurring decimal numbers are
irrational numbers
(vi)
Explain why square root and cube root of any non perfect square and non
perfect cube integer in not a rational number
4. Real Numbers
(i)
Definition of real numbers with examples
(ii)
Tell to students by default any numbers considered as real number until
and otherwise it is specially mention the type of numbers. This is the guide
line given by ETS.
(iii)
Explain Number tree
(iv)
Tell to students in general ETS do not use word Natural Number, Whole
Number, Rational Number and Irrational Number. They use positive integer

for natural number, non negative integer of whole number and fraction for
rational and irrational number.
5. Concept of odd and even integers
(i)
Define even and odd integer with examples
(ii)
Give general terms for even and odd integers in terms of integer N
(iii)
Explain why zero is an even number
(iv)
Give formula for sum of first N even positive number
(v)
Give formula for sum of first N odd positive number
(i)
2+4+6+ + (2N) = N(N+1)
(ii)
1+3+5+ ... +(2N-1) = N2

N N 1
2
(iii)

1+2+3+ . + N =

N N 1 (2 N 1)
6

(iv)
12 +22 +32 + N2 =
6. Concept of prime numbers
(i)
Define prime numbers with examples
(ii)
Let them identify only even prime number is 2
(iii)
Number of prime number in the given range such as 1-50 (Hint 15), 1-100
(Hint 25), 101-200 (Hint 21), 201-300 (Hint 17) and 1-1000 (Hint 168)
(iv)
Define co-prime with examples
(v)
Properties of prime numbers
(i)
Any prime number greater than 3 divided by 6, always leaves
remainder either 1 or 5 but converse may not be true.
(ii)
Square of any prime number greater than 3 divided by 12 or 24,
always leaves remainder 1
7. Composite Numbers
(i)
Define composite numbers with examples
(ii)
0 and 1 is neither prime nor composite.
(iii)
Explain why zero is not a composite number
8. Test of divisibility
(i)
Discuss test of divisibility condition for 2, 4, 8, 16
(ii)
Discuss test of divisibility condition for 3, 9
(iii)
Discuss test of divisibility condition for 5, 10
(iv)
Discuss test of divisibility condition for 11
(v)
Discuss test of divisibility condition for some composite numbers such as
6, 12, 15, 18 and 20
9. CONCEPT OF FACTORS
(i)
Define factor with example such as 1, 2, 3, 4, 6 and 12 are factors of 12.
Closer look, one and number itself is a factor for every numbers.
(ii)
Explain what is the significance of factor, I mean to say how 1, 2, 3, 4, 6
and 12 comes as factor of 12
(iii)
Give special formula to calculate factor for a given big number.

N a x .b y .c z .....

Note 1: Let any positive integer N and


Here a, b, c must be prime number
Number of positive integer factors for N would be (x+1)(y+1)(z+1)..
(i)
Explain logic to count total number of factors
(ii)
Explain logic to count total number of odd factors
(iii)
Explain logic to count total number of even factors
(iv)
Explain logic to count total number of different prime factors
(v)
Explain logic to count total number all possible prime factors

(vi)

10.LCM & HCF


(i)
(ii)
(iii)
(iv)
(v)
(vi)
(vii)
(i)
HCF
There

(ii)

Explain logic to count total number perfect square factors

Define factor once again


Explain Common Factor
Explain Highest Common factor (HCF)
Explain Multiple
Explain Common Multiple
Explain Least Common Multiple (LCM)
Let students know HCF is also known as GCD (Greatest Common Divisor)
are two ways to fined HCF
(i)
Factorization Method
(ii)
Long Division Method
(iii)
Explain both method with example
(iv)
Explain which method is suitable in which case

LCM (Least Common Multiple)


There are two ways to fined LCM
(i)
Factorization Method
(ii)
Long Division Method
(iii)
Explain both method with example
(iv)
Explain which method is suitable in which case

Examples: Official Guide 13th Edition


Chapter 5: Q2, 26, 32, 44,117
Chapter 6: Q7, 27

SESSION -1-CLASS ASSIGNMENT (HAND OUTS- MODULE-1ARITHMETIC)


1- What is the lowest positive integer that is divisible by each of the integers 1 through 7,
inclusive?
(A) 420
(B) 840
(C) 1260
(D) 2520
(E) 5040
2- If the sum of five consecutive positive integers is A, then the sum of the next five
consecutive integers in terms of A is:
(A) A+1
(B) A+5
(C) A+25
(D) 2A
(E) 5A
3- The sum of the even numbers between 1 and k is 79*80, where k is an odd number,
then k=?
(A) 79
(B) 80
(C) 81
(D) 157
(E) 159
4- AB + CD = AAA, where AB and CD are two-digit numbers and AAA is a three digit
number; A, B, C, and D are distinct digit. In the addition problem above, what is the
value of C?
(A) 1
(B) 3
(C) 7
(D) 9
(E) Cannot be determined
5- If s and t are positive integer such that s/t=13.12, which of the following could be the
remainder when s is divided by t?
(A) 2
(B) 10
(C) 13
(D) 19
(E) 45
6- How many positive integers less than 10,000 are such that the product of their digits is
210?
(A) 24
(B) 30
(C) 48
(D) 54
(E) 72

7- If x is an integer and 9<x2<99, then what is the value of maximum possible value of x
minus minimum possible value of x?
A. 5
B. 6
C. 7
D. 18
E. 20

8- The numbers {1, 3, 6, 7, 7, 7} are used to form three 2-digit numbers. If the sum of
these three numbers is a prime number p, what is the largest possible value of p?
A. 97
B. 151
C. 209
D. 211
E. 219
9- If n is a positive integer and p is a prime number, is p a factor of n!?
(1) p is a factor of (n+2)!-n!
(2) p is a factor of (n+2)!/n!
10-If p is a positive integer, what is the remainder when p 2 is divided by 12?
(1) p is greater than 3.
(2) p is a prime.
11-If a and b are integers and a.b=2, is a=2?
(1) b+3 is not a prime number
(2) a>b
12-If a, b and c are integers, is a.b.c an even integer?
(1) b is halfway between a and c
(2) a = b c
13-If x and y are integers, is x a positive integer?
(1) x*|y| is a prime number.
(2) x*|y| is non-negative integer.
14-If 6a=3b=7c, what is the value of a+ b+ c?
(1) ac=6b
(2) 5b=8a+4c
15-If x and y are integers, is y an even integer?
(1) 4y2+3x2=x4+y4
(2) y=4-x2

Answer Key
1. E
2. C
3. E
4. D
5. E
6. B
7. D
8. D
9. C
10.C
11.E
12.B
13.A
14.B
15.A

Homework Assignment: Official Guide 12th Edition (Given as CD with 13th Edition)
Number System
Problem solving:
1,2,3,6,7,8,11,15,22,23,24,26,28,29,30,32,35,36,37,38,40,42,43,44,45,46,50,51,54,56,58,59,
60,63,65,68,
70,72,73,74,75,77,79,81,82,83,87,89,90,91,95,98,100,104,106,107,108,110,112,114,117,120
,122,127,
129,133,135,136,137,138,140,141,142,143,148,150,155,159,161,163,164,165,169,172,175,1
76,181,182
185,186,188,190,195,196,198,200,203,204,208,211,213,216,217,218,219,220,224,225,226,2
27,230.
Data sufficiency:
1,5,6,8,9,12,13,15,16,17,22,23,24,25,26,27,30,31,32,35,36,40,41,43,44,46,51,54,57,60,61,62
,64,65,66,
69,70,72,73,76,82,83,85,86,90,91,95,96,98,99,100,101,103,106,108,110,115,118,119,120,12
1,123,125
128,130,133,137,138,139,150,151,152,153,154,155,156,158,161,162,165,167,168,170,171,1
72,174.

SESSION -2 (MODULE-1-ARITHMETIC)
Topics to be covered
Cyclist
Remainder
Percentages
Profit, Loss and Discount
1. CONCEPT OF UNIT DIGIT
(i)

Explain unit digit in a given number specially when given number is in


form of exponent.
(ii)
Show the process of finding unit digit in expansion of
(i)
2N
(ii)
3N
(iii)
4N
(iv)
5N
(v)
6N
(vi)
7N
(vii) 9N
To do these use the concept of cyclist process
2. REMAINDER OF REMAINDER
(I)
(II)
(III)
(IV)
(V)
(VI)

Explain Remainder with example


Explain formulae (Dividend = Divisor x Quotient +Remainder)
Explain how the product of decimal part of quotient and divisor is equal to
remainder.
Let them understand remainder cannot exceed divisor
Let them know remainder cannot be negative in general
Let them know remainder is always is always lesser than divisor

p
17.24
q
(iii)

Note1: If
where p and q are positive integer, remainder would be
0.24*q when p is being divided by q.
Give quick idea about remainder theorem
Note2: Remainder theorem tells either we perform operations (addition,
subtraction, Multiplication or exponent) then divide or first divide then
perforation operations (addition, subtraction, Multiplication or exponent)
remainder would be the same in both cases.
Example:

89
7

Remainder in the expression


is 2. General approach perform
operation multiplication 8*9=72 and 72 divided by 7 remainder is 2.

8 9 1 2
2

remainder 2
7
7
7

By using remainder theorem:


Here division performed first. I.e. if 8 and 9 divided by 7 the remainder are
1 and 2 are respectively.Work Out Examples
Two different positive integer x and y divided by 6 leaves remainders 3 and two
respectively. Find remainder when

1. x y is divided by 6
2. x y is divided by 6
3. y x is divided by 6
4. x y is divided by 6
5. x4 is divided by 6
6. y4 is divided by 6

3.

PERCENTAGES
Define percentage with basic information such as fraction whose denominator
is 100. Percent means per 100, out of 100, or divided by 100. For
example, 25% = 25/100 = 0.25 and 0.3% = 0.3/100 = 0.003. In terms of
money, 50 cents out of a dollar is 50 cents out of 100, which is 50/100 of a
dollar or 50% of a dollar.
(ii)
Explain how to find percentage
Note1:
To find a percentage of something, the percents must be converted
to decimals and then multiplied by some number.
Note2:
Never directly add and/or subtract percents; you must first multiply
them by something.
(i)

(iii)

To convert a decimal to a percent, move the decimal point two places from
left to right and add a % sign. For example, 0.8 = 80% and 0.02 = 2%.

(iv)

To convert a percent to a fraction, just make the percent the numerator of a


fraction with denominator 100 and reduce the fraction. For example, 40% =
40/100 = 4/10 = 2/5.

(v)

To convert a fraction to a percent, divide the numerator by the denominator


and move the decimal point two places to the right.
Use the following is a list of percents and their fraction and decimal
equivalents which should be committed to memory:
(i)
0.01= 1/100= 1%
(ii)
0.1= 1/10= 10%
(iii)
0.2= 1/5= 20%
(iv)
0.25= 1/4= 25%
(v)
0.33=1/3=33.33%
(vi)
0.4=2/5=40%
(vii) 0.5=1/2=50%

(vi)

(viii)
(ix)
(x)
(vi)

0.6=3/5=60%
0.75=3/4=75%
0.8=4/5=80%

How To Calculate Percentage Change Between Two Values


If the question is like that x is what percent of y, our answer would be

y 100 %

. Which is a correct but let student know

Value ( x )

100 %
Base ( y )

It is very important to understand what VALUE is and what


is BASE in the question being asked.
(vii)

Discuss Net percentage change in successive percentage changes such as


(i)
Increase by some percentage again increased by some other or same
percentage.
(ii)
Increase by some percentage again decrease by some other or same
percentage.
(iii)
Decrease by some percentage again increased by some other or same
percentage.
(iv)
Decrease by some percentage again decrease by some other or same
percentage.

( x ) ( y ) ( x ).( y )
%
100

Net Percentage change

Note:
This is the special formulae which include all four cases mentioned above. I am
sure all of us are aware of this special formula. For any clarification please
contact me.

4. PROFIT LOSS AND DISCOUNT


(i)

Discuss the terms related to profit loss and discount


(i)
CP (Cost Price)
(ii)
SP (Selling Price)
(iii)
MRP (Max Retail Price)
(iv)
MP (Market Price)
(v)
LP (List Price)
(vi)
Briefly explain the difference between MRP and MP
(vii) Briefly explain why LP
(viii) Markup
(ix)
Revenue
(x)
Profit
(xi)
Margin
(xii) Loss
(xiii) Discount
(xiv) Successive Discount
(xv) Why Margin

100 Pr ofit %

100

SP CP
(ii)

Discuss

100 loss %

100

SP CP
(iii)

Discuss

100 Pr ofit % 100 Pr ofit %

100
100

SP CP
(iv)

Discuss
profit made in some same transaction)

100 loss % 100 loss %

100
100

(This is in case of successive

SP CP
(v)

Discuss
made in some same transaction)

(This is in case of successive loss

100 Discount %

100

SP MP
(vi)

Discuss

100 Discount % 100 Discount %

100
100

SP MP
(vii)

Discuss
(This is in case of
successive discount made in some same transaction)

3. SIMPLE INTEREST AND COMPOUND INTEREST


(i)
Define what is Interest
(ii)
Define Simple Interest
(iii)
Define Compound Interest
Examples: Official Guide 13th Edition
Chapter 5: Q 11, 58
Chapter 6: Q2, 55, 62

SESSION -2-CLASS ASSIGNMENT (HAND OUTS- MODULE-1ARITHMETIC)


1) Find the unit digit or last digit of (782)87?
A) 2
B) 4
C) 8
D) 6
E) 1
2) What is unit digit of 6666 5555
A) 1
B) 2
C) 3
D) 4
E) 5
3) What is the units digit of (13)4 (17)2 (29)3?
A) 1
B) 2
C) 3
D) 4

E) 5
4) A chemist has 10 liters of a solution that is 10 percent nitric acid by volume. He wants to
dilute the solution to 4 percent strength by adding water. How many liters of water must
he add?
A) 5
B) 10
C) 15
D) 20
E) 12
5) An insurance company provides coverage for a certain dental procedure according to the
following rules: the policy pays 80% of the first $1,200 of cost and 50% of the cost above
$1,200. If a patient had to pay $490 of the cost for this procedure himself, how much did
the procedure cost?
A) 1200
B) 1500
C) 1700
D) 2000
E) 1900
6) The organizers of a fair projected a 25 percent increase in attendance this year over that
of last year, but attendance this year actually decreased by 20 percent. What percent of
the projected attendance was the actual attendance?
A) 32%
B) 64%
C) 45%
D) 50%
E) 25%
7) If 18 is 15 percent of 30 percent of certain number, what is the number?
A) 200
B) 400
C) 800
D) 1000
E) 500
8) If Mel saved more than $10 by purchasing a sweater at 15 percent discount, what is the
smallest amount the original price of the sweater could be, to the nearest dollar?
A) 60
B) 75
C) 67
D) 80
E) 90
9) If 25 percent of p is equal to 10 percent of q, and pq = 0, then p is what percent of q?
A) 10%
B) 20%
C) 30%
D) 40%
E) 50%
10) Positive integers y is 50 percent of 50 percent of positive integer x, and y percent of x
equals 100. What is the value of x?
A) 200
B) 600
C) 300
D) 500
E) 700
11) In country C, the unemployed rate among construction workers dropped from 16
percent on September 1, 1992, to 9 percent on September 1, 1996. If the number of
construction workers was 20 percent greater on September 1, 1996, than on September

1, 1992, what was the approximate percent change in the number of unemployed
construction workers over this period?
(A) 50% decrease
(B) 30% decrease
(C) 15% decrease
(D)30% increase
(E) 55% increase
12) Marys income is 60 percent more than Tims income, and Tims income is 40 percent
less than Juans income. What percent of Juans income is Marys income?
A) 50%
B) 70%
C) 40%
D) 80%
E) 96%
13) A factory has 500 workers, 15 percent of whom are women. If 50 additional workers are
to be hired and all of the present workers remain, how many of the additional workers
must be women in order to raise the percent of women employees to 20 percent?
A) 25%
B) 35%
C) 50%
D) 60%
E) 65%

14) The cost price of 20 articles is same as the selling price of x articles. If the profit is 25%,
then the value of x is?
A) 12
B) 14
C) 16
D) 20
E) 24
15) In certain store, the profit is 320% of the cost. If the cost increase by 25% but the
selling price remains constant, approximately what percentage of the selling price is
profit?
A) 70
B) 30
C) 100
D) 150
E) 250
16) A jewelry dealer initially offered a bracelet for sale at an asking price that would give a
profit to the dealer of 40 percent of the original cost. What was the original cost of the
bracelet?
(1) After reducing this asking price by 10 percent, the jewelry dealer sold the bracelet at
a profit of $403.
(2) The jewelry dealer sold the bracelet for $1,953.
17) In a certain packinghouse, grapefruit are packed in bags and the bags are packed in
cases. How many grapefruit are in each case that is packed?
(1) The grapefruit are always packed 5 to a bag and the bags are always packed 8 to a
case.
(2) Each case is always 80 percent full.
18) If the price of a magazine is to be doubled, by what percent will the number of
magazines sold decrease?

(1) The current price of the magazine is $1.00.


(2) For every $0.25 of increase in price, the number of magazines sold will decrease by
10 percent of the number sold at the current price.
19) For a certain bottle and cork, what is the price of the cork?
(1) The combined price of the bottle and the cork is 95 cents.
(2) The price of the bottle is 75 cents more than the price of the cork.
20) Last year an employee received a gross annual salary of $18,000, which was paid in
equal paychecks throughout the year. What was the gross salary received in each of the
paychecks?
(1) The employee received a total of 24 paychecks during the year.
(2) The employee received a paycheck twice a month each month during the year.

Answer Key
1. C
2. A
3. A
4. C
5. C
6. B
7. B
8. C
9. D
10.A
11.B
12.E
13.B
14.C
15.A
16.A
17.A
18.C
19.C
20.D
Homework Assignment:
OG 12
Problem solving: 10, 13, 17, 19, 47, 55, 64, 84, 92, 94, 111, 115, 123, 131, 139, 151, 187, 202
Data sufficiency: 2, 7, 33, 37, 52, 55, 63, 77, 88, 142, 143

SESSION-3 (MODULE-1-ARITHMETIC)
Topics to be covered
Ratios
Proportions & Mixture
Averages

1. RATIO
(i)
Define Ratios
Note: Ratios, like fractions, decimals, and percents, are just another way of
expressing division. Every fraction is a ratio and every ratio is a fraction. A
fraction is just the ratio of the numerator to the denominator. The ratio 1: 2
(read 1 to 2) is equivalent to the fraction 1/2 or the decimal 0.5 or 50% or
just 1 divided by 2. On the GRE, ratios may be expressed in any of the
following ways:

(i)
x/y
(ii)
the ratio of x to y
(iii)
x is to y
(iv)
x:y
(ii)
Anytime you see a ratio, treat it just like a fraction. Anything you can do to a
fraction, you can also do to a ratio, including cross-multiplying, reducing,
finding common denominators, etc.
(iii)
Let students know ratio can be given only for similar units variable or numbers
(iv)
Properties of Ratio
(i)
Any ratio multiplied by a constant term ratio remains unchanged
(ii)
Any ratio divided by any non zero constant term ratio remains unchanged
(iii)
Any ratio added by a constant term ratio may changed
(iv)
Give tips how to club two individual ratio to make a combine ratio
Hint1: If a:b and b:c is given how to find a:b:c
2. PROPORTION
(i)
Define Proportion
Note: The GRE often contains questions in which you must compare two
ratios which are proportional. These questions take a given ratio, or
relationship, and project it onto a larger or smaller scale while leaving
out one piece of information.
Example: If 10 baskets contain a total of 50 eggs, how many eggs
would 7 baskets contain? (a) 10 (b) 17 (c) 35 (d) 40 (e) 50

(ii)

Explain all kinds of proportions properties such as rule of CD


(COMPONENDO AND DIVIDEND)
Quick review of continued proportion
Quick review of mean proportion
Explain the logic behind proportion, I mean why we use proportions.

(iii)
(iv)
(v)
3. VARIATIONS
(i)
Define variation with some real life examples.
(ii)
Explain all kinds of variations
(i)
Direct variation with real life examples
(ii)
Inverse variation with real life examples
(iii)
Joint variation with real life examples
(iv)
Explain why this is known as joint variations
(v)
Show the importance of variation applications
(i)
Concept of variation can be used in Time Speed and Distance
problems
(ii)
Concept of variation can be used in Time and work problems
(iii)
Explain how this is related to rate concept
4. AVERAGES
(i)

(ii)

(iii)

Define Averages with real life examples


Note: In mathematics, an average is a measure of the "middle" or "typical"
value of a data set
Give more info about average
Note: In the most common case, the data set is a list of numbers. The
average of a list of numbers is a single number intended to typify the
numbers in the list. If all the numbers in the list are the same, then this
number should be used. If the numbers are not the same, the average is
calculated by combining the numbers from the list in a specific way and
computing a single number as being the average of the list.
Just give the idea to students in GRE testing average and arithmetic mean
both is same
Note: Arithmetic mean

Average

(iv)
(v)

Sum of observations
Total Number of observations

The mean is neither less than the minimum value nor greater than the
maximum value of given observations.
Explain how average effect if each observation is..
(i)
Increased by any constant term k
(ii)
Decrease by any constant term k
(iii)
Multiplied by any constant term k
(iv)
Divided by any constant term k
(v)
Give complete clarity about combined average which is very
important. It is not that we are giving just formulae let them have
clear view on this. (The arithmetic mean of several sets of data may
be combined into a single arithmetic mean for the combined sets of
data)
Examples: OG 13th Edition
Chapter 5 : Q16, 30, 113
Chapter 6: Q81, 153

SESSION-3-CLASS ASSIGNMENT (HAND OUTS-MODULE-1ARITHMETIC)


1) The weight of every type A widget is same, and The weight of every type B widget is
same, and The weight of every type C widget is same. If the weight of 8 types A widget
is equal to the weight of 3 type B widget, and the weight of 5 type B widget is equal to
the weight of 7 type C widget. What is the ratio of the total weight of 1 type widget A
and 1 type widget B to the total weight of 1 type B widget and 1 type C widget? (Ans.
D)
A) 75/97
B) 25/21
C) 16/15
D) 77/96
E) 96/77
2) At a certain school, the ratio of the number of second graders to the number of fourth
graders is 8 to 5, and, the ratio of the number of first graders to the number of second
graders is 3 to 4. If the ratio of the number of third graders to the number of fourth
graders is 3 to 2. What is the ratio of the number of first graders to the number of third
graders? (Ans. A)
A) 4:5
B) 5:4
C) 2:3
D) 1:2
E) 12:7
3) The ratio of marketing to finance majors at Empire Business School is 5:8. The ratio of
finance to accounting majors is 3:2. If there are 240 marketing majors, how many more
accounting majors than marketing majors are there? (Ans. E)
A) 20
B) 12
C) 24
D) 25
E) 16
4) The ratio, by volume, of soap to alcohol to water in a certain solution is 2:50:100. The
solution will be altered so that the ratio of soap to alcohol is doubled while the ratio of
soap to water is halved. If the altered solution will contain 100 cubic centimeter of
alcohol, how many cubic centimeters of water will it contain? (Ans. B)
A) 400
B) 800

C) 1000
D) 1200
E) 1500
5) A crate of apples contains 1 bruised apple for every 30 apples in the crate. Three out of
every 4 bruised apples are considered not fit to sell, and every apple that is not fit to
sale bruised. If there are 12 apples not fit to sell in the crate, how many apples are
there in the crate? (Ans. C)
A) 400
B) 500
C) 480
D) 640
E) 740

6) A container holding 12 ounce of a solution that is 1 part alcohol to 2 parts water is


added to a container holding 8 ounces of a solution that is 1 part alcohol to 3 part
water. What is the ratio of alcohol to water in the resulting solution? (Ans. A)
A) 3:7
B) 10:3
C) 5:2
D) 12:5
E) 2:3
7) If

1
2

of the number of white mice in a certain laboratory is

of mice, and

1
3

of the number of gray mice is

1
9

1
8

of the total number

of the total number of mice, then

what is the ratio of the number of white mice to number of gray mice? (Ans. E)
A) 4:3
B) 2:3
C) 3:2
D) 5:7
E) 3:4
8) A certain company that sells only cars and trucks reported that revenues from car sales
in 1997 were down 11 percent from 1996 and revenues from truck sales in 1997 were
up by 7 percent from 1996. If total revenues from car sales and truck sales in 1997
were up 1 percent from 1996, what is the ratio of revenue from car sales in 1996 to
revenue from truck sales in 1996? (Ans. A)
A) 1:2
B) 5:3
C) 3:5
D) 2:1
E) 1:4
9) In a certain district, the ratio of the number of registered Republicans to the number of

registered Democrats was 5 . After 600 additional Republicans and 500 additional

Democrats registered, the ratio was 5 . After these registrations, there was how many
more voters in the district registered as Democrats than as Republicans? (Ans.B)
A) 200
B) 300

C) 400
D) 500
E) 600
10)
Four staff members at a certain company worked on a project. The amounts of
time that the four staff members worked on the project were in the ratio 2 to 3 to 5 to
6. If one of the four staff members worked on the project for 30 hours, which of the
following CANNOT be the total number of hours that the four members worked on the
project? (Ans.D)
(A) 80
(B) 96
(C) 160
(D)192
(E) 240

11)
The integer n is greater than 7. The average of a group of n numbers is a. when
3 of the numbers are removed; the average of the remaining numbers is b. which of the
following expressions is the average of the 3 numbers removed? (Ans. B)
A)

na
n3

B)

n ( ab )+3 b
3

C)

n ( ab )+ 3 b
n3

D)

n ( a+b ) +3 b
3

E)

n(ab)
3

12)
There are x numbers in list L where x is a positive integer, and There are y
numbers in list M, where y is a positive integer. The average of numbers in list L is p
and the average of all the numbers in both list L and M is q. which of the following
expressions is the average of the numbers in list M? (Ans. A)
A)

( q p ) x +qy
y

B)

( q p ) x +qy
P

C)

( q p ) x +qy
q

D)

( q p ) p+qy
y

E)

( q p ) x +qy
pq

13)
List S consists of 10 consecutive odd integers, and list T consists of 5 consecutive
even integers. If the least integer in S is 7 more than the least integer in T, how much
greater is the average of the integer in S than the average of the integers in T? (Ans. B)
A) 14
B) 12
C) 10
D) 15
E) 20
14)
Last year Department X had a sales total for December that was 4 times the
average of the monthly sales total for January through November. The sales total for
December was what fraction of the sales total for the year? (Ans. A)
A) 4/15
B) 15/4
C) 2/5
D) 1/2
E) 5/3
15)
The average of the integers from 200 to 400, inclusive, is how much greater than
the average of the integers from 50 to 100, inclusive? (Ans. D)
A) 220
B) 230
C) 240
D) 225
E) 500
16)
If a rope is cut into three pieces of unequal length, what is the length of the
shortest of these pieces of rope?
(1) The combined length of the longer two pieces of rope is 12 meters.
(2) The combined length of the shorter two pieces of rope is 11 meters.

17)

Foo
d
S
T

Number
of
Calories
per
Kilogra
m
2,000
1,500

Number of
Grams
of Protein
per
Kilogram
150
90

The table above gives the number of calories and grams of protein per kilogram of
foods S and T. If a total of 7 kilograms of S and T are combined to make a certain food
mixture, how many kilograms of food S are in the mixture?
(1) The mixture has a total of 12,000 calories.
(2) The mixture has a total of 810 grams of protein.
18)
The cost to charter a certain airplane is x dollars. If the 25 members of a club
chartered the plane and shared the cost equally, what was the cost per member?
(1) If there had been 5 more members and all 30 had shared the cost equally, the
cost per member would have been $40 less.
(2) The cost per member was 10 percent less than the cost per person on a
regularly scheduled flight.
19)
Did United States carriers use more than 10 billion gallons of jet fuel during
1983?

(1) United States carriers paid a total of $9.4 billion for the jet fuel used in 1983.
(2) United States carriers paid an average (arithmetic mean) of $0.90 per gallon for
the jet fuel used in 1983.
20)

What was Bills average (arithmetic mean) grade for all of his courses?

(1) His grade in social studies was 75, and his grade in science was 75.
(2) His grade in mathematics was 95.

Answer Key
1. D
2. A
3. E
4. B
5. C
6. A
7. E
8. A
9. B
10.D
11.B
12.A
13.B
14.A
15.D
16.E
17.D
18.A
19.C
20.E

Homework Assignment: EGE GMAT Math Workbook Chapter 1

SESSION-4 (MODULE-1-ARITHMETIC)
Topics to be covered
Time, speed and Distance
Time & Work
1. RATE
(i)
(ii)
(iii)
(iv)

(TIME SPEED AND DISTANCE)


Explain concept of motion
Define speed with help of example
What are the units used to measure motion components like time speed and
distance?
Explain by taking real life example how distance is related to speed if time is
constant.

D1 S1

D2 S2
(v)

Hints
Explain by taking real life example how distance is related to time if speed is
constant.

D1 T1

D2 T2
(vi)

Hints
Explain by taking real life example how speed is related to time if distance is
constant.

T1 S2

T2 S1
(vii)

Hints
Explain by taking real life example how distance is related to speed and time.

D1 T1 S1

D2 T2 S2
(viii)

Hints
OR Distance = Time x Speed
Define average speed
(i)
Give fundamental formula to calculate average speed

Average Speed
(ii)
(iii)
(iv)
(viii)

(ix)
(x)

Total Dis tan ce Travel in Journey


Total Time Taken in Journey

Hints
Let students understand average speed does not mean that always
arithmetic mean of given different speeds
Explain if distance is constant how average speed can be calculated
Explain if time is constant how average speed can be calculated

Define Relative Speed


(i)
Take real life example such train or cars to understand relative speed
(ii)
Let them understand the different between speed and relative speed
(iii)
Relative speed if speeds of moving bodies are different
Hints Relative Speed = Sum or Difference of speeds
(iv)
Relative speed if speeds of moving bodies are same and moving in same
direction.
Hints Relative Speed = 0
(v)
Relative speed if of moving bodies are moving is same direction
Hints Relative Speed = Difference of speeds
(vi)
Relative speed if of moving bodies are moving is opposite direction
Hints Relative Speed = Sum
Explain Relative Speed in case of Circular Motions (Brief Idea)
Explain Relative Speed in case of boats and stream (Brief Idea)

2. RATE (TIME AND WORK)


(i)
(ii)

Give some clarity about concept time and work


Give more clarity on assumption made in concept time and work
Hints: Let us take an example 10 men can paint 10 walls in 10 days working 10
hours a day.
(1) Assumption1: All ten men are equally efficient
(2) Assumption2: All ten men are doing same amount of work every day which in
reality may not be possible
(3) Assumption3: All walls are identical until exclusively given they are different
in dimensions
(4) Assumption4: Every day work done by all men together are same

(iii)

Give individual relation between Work done and Men


Hint: Work done is directly proportional to Men

(iv)

Give individual relation between Work done and number of days spent on work
done. Hint: Work done is directly proportional to number of days spent on work

(v)

Give individual relation between Work done and number of hours work done in a
day. Hint: Work done is directly proportional to number of hours work done in a
day

(vi)

Give individual relation between Work done and efficiency of working people. It
could be Man, Women, Children or Boys, Machine and Pipes

Hint: Work done is directly proportional to efficiency of working people


(vii)
(viii)
(ix)

Give idea to students most of the cases work done are same but it may differ
also
Give very clear explanation to students why this concepts known as rate
Discuss Master Formula to solve most of time and work problem which include all
above relations

M 1. D1.T1. E1 M 2 .D2 .T2 .E2

W1
W2
Master Formula
MMan Power
DNumber of days
TNumber of hours
W Work done
EEfficiency of working people or objects (work rate)
(x)
Discuss how to find total number of days required finishing a task if two or three
people working together and their individual time required to finish the task is
given.
(xi)
Discuss how to find total number of hours required of filling or emptying a tank if
two or three pipes together filling or emptying a tank and their individual time
required to finish the task is given.
(xii) It is very important to discuss reciprocal of one day or one hour work is total time
required to finish the task.
Examples:
OG 13 Chapter 5 Q 81,103,119,139,144
Chapter 6 : Q 12, 68

SESSION-4-CLASS ASSIGNMENT (HAND OUTS-MODULE-1ARITHMETIC)


1. A, B, and C were to be paid in proportion to the part of work they did while working on
the same piece of work. A and B individually can finish the piece of work in 12 days
and 15 days respectively. They worked together for five days and then C completed the
remaining work all alone. If $720 was the net sum to be paid for the entire work, what
was the average daily wage of B?
(A) $144
(B) $90
(C) $60
(D) $54
(E) $48
Answer = E
2. Mr. Peter used to be late by 30 minutes every day to reach to his office. On a particular
day he reduce his speed by 25% hence late by 50minute. By what percent he has to
increase his speed in order to reach on time?
(A)
300%
(B)
150%
(C)
100%

(D)
(E)

50%
25%
Answer = C
3. At 12 pm a train left a station at 60 m/h. 60minute later another train left the same
station at 70 mile/h but in the opposite direction. At what time will the two trains be
710 miles apart?
(A) 5pm
(B) 6pm
(C) 7pm
(D)12am
(E) 1am
Answer = B
4. Fanny and Alexander are 360 miles apart and are traveling in a straight line toward
each other at a constant rate of 25 mph and 65 mph respectively, how far apart will
they be exactly 1.5 hours before they meet?
A. 25 miles
B. 65 miles
C. 70 miles
D. 90 miles
E. 135 miles
Answer = E
5. In racing over a given distance d at uniform speed, A can beat B by 20 yards, B can
beat C by 10 yards, and A can beat C by 28 yards. Find d, in yards.
A. 58
B. 72
C. 100
D. 116
E. 120
Answer = C
6. Zander drives to work at an average speed of 40 miles per hour and returns home
along the same route at an average speed of 24 miles per hour. If his total travel time is
4 hours, what is the total number of miles in the roundtrip to and from work?
(A) 48
(B) 60
(C) 96
(D) 120
(E) 144
Answer = D
7. Machine A produces 100 parts twice as fast as Machine B does. Machine B produces
100 parts in 40 minutes. If each machine produces parts at a constant rate, how many
parts does Machine A produce in 6 minutes?
(A) 30
(B) 25
(C) 20
(D) 15
(E) 7.5
Answer = A
8. Bottle X is 2/3 full of water, which is half the capacity of Bottle Y. Bottle Y is 3/4 as full of
water and has twice the capacity of bottle Z which is 1/4 full of water. If all the water
from both Bottle X and Bottle Z were poured into Bottle Y, Bottle Y would then be filled
to what fraction of its capacity?
(A) 29/24
(B) 7/8
(C) 5/6
(D) 3/4
(E) 2/3
Answer = C

9. A pool which was 2/3 full to begin with, was filled at a constant rate for 5/3 hours until it
was until it was 6/7 full. At these rates, how much time would it take to completely fill
this pool if it was empty to begin with?
A. 8 hrs 45 min
B. 9 hrs. 0 min
C. 9 hrs 30 min
D. 11 hrs 40 min
E. 15 hrs 30 min
Answer = A
10.Lindsay can paint 1/x of a certain room in one hour. If Lindsay and Joseph, working
together at their respective rates, can paint the room in one hour, what fraction of the
room can Joseph paint in 20 minutes?
A. 1/3x
B. x/(x-3)
C. (x-1)/3x
D. x/(x-1)
E. (x-1)/x
Answer = C
11. Carmen currently works 30 hours per week at her part-time job. If her gross hourly wage were to increase by $1.50,
how many fewer hours could she work per week and still earn the same gross weekly pay as before the increase?
(1) Her gross weekly pay is currently $225.00.
(2) An increase of $1.50 would represent an increase of 20 percent of her current gross hourly wage.
Answer = D
12. At what speed was a train traveling on a trip when it had completed half of the total distance of the trip?
(1) The trip was 460 miles long and took 4 hours to complete.
(2) The train traveled at an average rate of 115 miles per hour on the trip.
Answer = E
1

1
2

13. A certain employee is paid $6 per hour for an 8-hour workday. If the employee is paid
times this rate for time
worked in excess of 8 hours during a single day, how many hours did the employee work today?
(1) The employee was paid $18 more for hours worked today than for hours worked yesterday.
(2) Yesterday the employee worked 8 hours.
Answer = C
14. Chan and Micko drove separate cars along the entire length of a certain route. If Chan made the trip in 15 minutes,
how many minutes did it take Micko to make the same trip?
3
4
(1) Mickos average speed for the trip was
(2) The route is 14 miles long.
Answer = A

of Chans average speed.

15. Water is pumped into a partially filled tank at a constant rate through an inlet pipe. At the same time, water is
pumped out of the tank at a constant rate through an outlet pipe. At what rate, in gallons per minute, is the amount
of water in the tank increasing?
(1) The amount of water initially in the tank is 200 gallons.
(2) Water is pumped into the tank at a rate of 10 gallons per minute and out of the tank at a rate of 10 gallons every
1
2
2
minute.
Answer = B

Homework assignment:
EGE Workbook Chapter 5 -Time, Speed, Distance & Time & Work

SESSION-5 (MODULE-2-ALGEBRA)
Topics to be covered
Indices
Linear Equations
Simultaneous Equations
Quadratic Equations
Quadratic Expression
CONCEPT TO BE TESTED IN ALGEBRA
(i)

Exponents (Indices and Surds) and Linear Equations and Quadratic


Equations (Word Problems)
(ii)
Inequalities and Absolute Values
(iii)
Sequences and Set Theory and Its Applications
(iv)
Function and Defined functions application of algebra
1. INDICES AND SURDS
(i)
Before you begin the concept just brush up basic algebra formulae

(ii)
(iii)
(iv)
(v)
(vi)
(vii)

Define Indices and Surds with examples


Explain the difference between indices and surds
Let students know that in GRE testing most of the cases ETS Tests
question from indices.
Explain Index Notations
Index Notations Negative Base
Index Notations Laws
Hints

Note 1:

If a x a y
Then x y ; provided a 1
Because

13 15 but 3 5

Note 2 :
If a x b x
Then a b ; provided x 0
Because

30 50 but 3 5

2. POLYNOMIALS:
(i)
Just give brief idea about polynomial
Hints: A polynomial is an expression having terms with decreasing powers of x,
like this: 2x3 + 3x2 - x + 6. Polynomial is an equation of constants and only
variable with different degree. ax n+bxn-1+cxn-2+dxn-3+ + k =0
Note1: nth order polynomial will have n roots (roots means value of variable
which satisfy given equation)

3. LINEAR EQUATIONS:
(i)
Define Equations
(ii)
Definition of simple equation
(iii)
Degree of equation
(iv)
Method to solve equations
(v)
Definition of linear equation
(vi)
Explanatory examples to explain the above concepts
(vii) Definition of independent equation, dependent equation, quadratic
equation, linear equation in two variables Introduction to the concept of
system of linear equations and the two methods to solve simultaneous
linear equations, namely, solving by substitution and solving by
elimination
(viii) Different types of solutions to a system of linear equations
(i)
Unique Solutions (with example)
(ii)
No Solution (with example)
(iii)
Infinite Solution (with example)
(ix)
Special cases in linear equations
(x)
Discussion of word problems.
4. QUADRATIC EQUATIONS:
(i)
Define quadratic equation
Hints: Quadratic Equations or equations of the second order where the
variables have a power of 2 is tested. Solutions, roots, types of roots of
quadratic equations are tested. Factorizing a quadratic equation to find its
solutions is also tested in GRE. Introduction to quadratic equations and
roots of quadratic equations

Note: Let students know for some cases this known as binomial
Concept of discriminant of quadratic equation
Method to calculate the sum, difference and product of roots.
Method to form a quadratic equation, if roots are given.
Method to determine nature of roots (real, rational, equal, imaginary),
using value of the discriminant
5. DISCUSS QUADRATIC EXPRESSION
(i)
Explain what is quadratic expression
(ii)
Explain the difference between quadratic equation and
quadratic equation
(iii)
Let students know, now a day GRE testing this type of
question also.
(iv)
Give formula to find minimum and maximum value of a
given quadratic expression.
Hints:
(xi)
(xii)
(xiii)
(xiv)

f ( x ) ax 2 bx c
Case 1: a 0
f ( x ) Max
4ac b 2
4a
Case 1: a 0
f ( x ) Min

4ac b 2
f ( x ) Max
4a
f ( x ) Min

Examples: OG 13th Edition


Chapter 5: 37
Chapter 6: Q86, 99, 132

SESSION-5-CLASS ASSIGNMENT (HAND OUTS-MODULE-2-ALGEBRA)


1- This year Trek will save a certain amount of his income, and he will spend the rest. Next
year Trek will have no income, but for each dollar that he saves this year, he will have 1
+ r dollars available to spend. In terms of r, what fraction of his income should Trek
save this year so that next year the amount available to spend will be equal to half the
amount that he spends this year?
(A) 1/(r+2)
(B) 1/(2r+2)
(C) 1/(3r+2)
(D) 1/(r+3)
(E) 1/(2r+3)
Answer = E
2- The price of a bushel of corn is currently $3.20, and the price of a peck of wheat is
$5.80. The price of corn is increasing at a constant rate of 5.x cents per day while the

x. 2 x

price of wheat is decreasing at a constant rate of


cents per day where number
of is days. What is the approximate price when a bushel of corn costs the same amount
as a peck of wheat?
(A) $4.50

(B) $5.10
(C) $5.30
(D) $5.50
(E) $5.60
Answer E
3- In an ensemble of gongs, all gongs have a diameter of either ten inches, or twelve
inches or fifteen inches. In the collection there are 18 ten inch gongs. Half of the gongs
in the collection are Tiger gongs. Of the Tiger gongs, there are equal numbers of ten
inch, twelve inch and fifteen inch gongs. Half of the twelve inch gongs are not Tiger
gongs, and half of all gongs are fifteen inches in diameter. How many gongs are there
in the collection?
A. 18
B. 54
C. 72
D. 90
E. 108
Answer: E.
4- The rate of a certain chemical reaction is directly proportional to the square of the
concentration of chemical A present and inversely proportional to the concentration of
chemical B present. If the concentration of chemical B is increased by 100%, which of
the following is closest to the percent change in the concentration of chemical A
required to keep the reaction rate unchanged?
A. 100% decrease
B. 50% decrease
C. 40% decrease
D. 40% increase
E. 50% increase
Answer: D.
5- A certain city with population of 132,000 is to be divided into 11 voting districts, and no
district is to have a population that is more than 10 percent greater than the population
of any other district. What is the minimum possible population that the least populated
district could have?
A. 10,700
B. 10,800
C. 10,900
D. 11,000
E. 11,100
Answer: D

6- Eight litres are drawn off from a vessel full of water and substituted by pure milk. Again
eight litres of the mixture are drawn off and substituted by pure milk. If the vessel now
contains water and milk in the ratio 9:40, find the capacity of the vessel.
A. 21 litres
B. 22 litres
C. 20 litres
D. 14 litres
E. 28 litres
Answer D
7- If b is an integer, is
1)
is an integer
2)

an integer?

Answer = B
8- If a and b are positive integers what is the value of a+b?
1) a/b = 5/8
2) The greatest common divisor of a and b is 1
Answer is C.
9- What is the remainder when the positive integer n is divided by the positive integer k,
where k>1?
1) n = (k+1)3
2) k = 5
Answer Is A.
10-Question 3: If there are more than 2 numbers in a certain list, is each of the numbers in
the list equal to 0?
1) The PRODUCT of any 2 numbers in the list is equal to 0
2) The SUM of any 2 numbers in the list is equal to 0
Answer is B.
11-Is y an integer?
(1) y3 is an integer
(2) 3y is an integer
Answer = C
12-Of the 58 patients of Vertigo Hospital, 45 have arachnophobia. How many of the
patients have acrophobia?
(1) The number of patients of Vertigo Hospital who have both arachnophobia and
acrophobia is the same as the number of patients who have neither arachnophobia nor
acrophobia.
(2) 32 patients of Vertigo Hospital have arachnophobia but not acrophobia.
Answer: A.
13-A certain fruit stand sold total of 76 oranges to 19 customers. How many of them
bought only one orange?
(1) None of the customers bought more than 4 oranges
(2) The difference between the numbers of oranges bought by any two customers is
even
Answer: D.
14-There is at least one viper and at least one cobra in Pandora's box. How many cobras
are there?
(1) There are total 99 snakes in Pandora's box.
(2) From any two snakes from Pandora's box at least one is a viper.
Answer: B.
15-Linda has $15, which is enough to buy 11 muffins and 7 brownies, is $45 enough to
buy 27 muffins and 27 brownies?
(1) $15 is enough to buy 7 muffins and 11 brownies.
(2) $15 is enough to buy 10 muffins and 8 brownies.
Answer: D

Answer Key
1. E
2. E
3. E
4. D
5. D
6. D
7. B
8. C
9. A
10.B
11.C
12.A
13.D
14.B
15.D
Homework Assignment: EGE Workbook
Chapter 2 Q1-26 Q41-49

SESSION-6-(MODULE-2-ALGEBRA)
Topics to be covered
Inequalities
Absolute Value
Sequences
1. NUMBER LINE
(i)
(ii)
(i)

(ii)

Explain Number Line


Explain most important aspect of line
Numbers on number line are in ascending order only. All left side
number are always lesser than right side number irrespective of their
sign
Hint: 10>6 but (-10)<(-6)
All numbers on number line are real number

(iii)
(iv)

Any number right side to zero are positive and left to zero are always
are negative
Zero is neither positive nor negative number

2. INEQUALITIES (basics)
(i)
Define Inequalities
Hint: An inequality is comparison between or amongst two statements with different
values.

(ii)
(iii)
(iv)
(v)
(vi)
(vii)

3.

Define Linear Inequalities


Define Quadratic Inequalities
Explain how inequalities differs from equation
Explain what is finite solution of any linear in-equation
Explain what is infinite solution of any linear in-equation
Properties of inequalities
(i)
How inequality changes if a positive number added both side
(ii)
How inequality changes if a positive number subtracted both
side
(iii)
How inequality changes if a positive number multiplied both
side
(iv)
How inequality changes if a positive number divided both side
(v)
How inequality changes if a negative number added both side
(vi)
How inequality changes if a negative number subtracted both
side
(vii) How inequality changes if a negative number multiplied both
side
(viii) How inequality changes if a negative number divided both side
INEQUALITIES (Fundamental)
(i)
Explain the meaning of strictly lesser inequality
(ii)
Explain the meaning of lesser inequality
(iii)
Explain the meaning of strictly greater inequality
(iv)
Explain the meaning of greater inequality
(v)
Discuss how to solve and write the answer of strictly lesser
linear in-equation
(vi)
Discuss how to solve and write the answer of lesser linear inequation
(vii) Discuss how to solve and write the answer of strictly greater
linear in-equation
(viii) Discuss how to solve and write the answer of greater linear inequation
(ix)
Discuss how to solve and write the answer of strictly quadratic
in-equation
(x)
Discuss how to solve and write the answer of lesser quadratic
in-equation
(xi)
Discuss how to solve and write the answer of strictly greater
quadratic in-equation
(xii) Discuss how to solve and write the answer of greater quadratic
in-equation

4. INEQUALITIES (Short-cuts for quadratic Inequality)

x is var iable a, b are any real cons tan t numbers


Case 1
If ( x a )( x b) 0 and it is given a b
Then solution is...
x ( a , b)
Case 2
If ( x a )( x b) 0 and it is given a b
Then solution is...
x a,

Case 3
If ( x a )( x b) 0 and it is given a b
Then solution is...
x ( ,

a ) (b,

Case 4
If ( x a )( x b) 0 and it is given a b
Then solution is...
x ,

a b,

5. ABSOLUTE VALUES
(i)

(ii)

Give mathematical definition of absolute values


The absolute value of x equals
(i)

x when x is greater than zero

(ii)

0 when x equals 0

(iii)

-x when x is less than zero (this "flips" the number back to positive)

Absolute of any variable are number cannot be negative i.e. the


minimum value of any absolute is zero.
Absolute Value means...only how far a number is from zero:

"6" is 6 away from zero,


and "-6" is also 6 away from zero.
So the absolute value of 6 is 6,
and the absolute value of -6 is also 6
Example: Absolute Value Symbol To show you want the absolute value of
something, you put "|" marks either side (called "bars"), like these examples:
|7| = 7
|-5| = 5

(iii)
(iv)
(v)
(vi)
(vii)
(viii)
(ix)
(x)
(xi)
(xii)
(xiii)
(xiv)

Discuss the properties of absolute values


Lets students understand how inequalities and absolutes values are
closely related.
Let students know the solution of IxI=-3 is not defined because absolute
of anything cannot be negative
Let students know the solution of IxI<-3 is not defined because absolute
x 3
of anything cannot be negative
Let students know the solution of
is not defined because absolute
x 3
of anything cannot be negative
Let students know the solution of
is real number because
absolute of anything cannot be negative which means all values will be
x 3
greater than -3
Let students know the solution of
is real number because
absolute of anything cannot be negative which means all values will be
x a
x2 a
greater than -3
x

a
Let students know
means a x a
2
a x a
Let students know x ameans
2
Let students knows x a means a x a
Let students knows
means
Explain how to solve absolute values related problems in different cases

Case 1

If x k then Solution is... x k or x k


Case 2
If x k then Solution is ...
x (k ,
Case 3

k)

If x k then Solution is ...


x k ,

Case 4
If x k then solution is...
x ( ,
Case 5

a ) (b,

If If x k then solution is...


x ,

a b,

Case 6
If x y then solution is...
x y

or x y

6. SEQUENCE
(i)
Define Sequences
Hints: This covers simple and the typical series including arithmetic progression and
geometric or multiplicative progression and Harmonic progressions with formulae to find
the nth term of progression and the sum of n terms of progression.

(ii)
(iii)
(i)
(ii)

Let students know that some of the cases ETS tests user defined sequence
Hint: A user defined sequence is they will give relations from one term to
another and then they frame questions.
Let students know that some of the cases ETS tests standard well defined
sequence
Arithmetic sequence
Geometric Sequence

(v)

Give closer look of sequences to students


(i)
The number patterns
(ii)
Sequences of numbers
(iii)
Arithmetic sequences
Hints: In arithmetic sequence next term sequence is always increase or decrease
by a constant term from its previous term and this constant term is known as
common difference. A finite portion of an arithmetic progression is called a finite
arithmetic progression and sometimes just called an arithmetic progression.

(vi)

(vii)

Discuss the sum arithmetic sequence


Geometric sequences
Hints: In geometric sequence next term of the sequence is constant times

of previous term. This constant time is known as common ratio


Properties:
(i) Positive, the terms will all be the same sign as the initial term.
(ii) Negative, the terms will alternate between positive and negative.
(iii)Greater than 1, there will be exponential growth towards positive
infinity.
(iv)
1, the progression is a constant sequence.
(v) Between 1 and 1 but not zero, there will be exponential decay
towards zero.
(vi)
1, the progression is an alternating sequence (see alternating
series)
(vii) Less than 1, for the absolute values there is exponential growth
towards positive and negative infinity (due to the alternating sign).
NOTE: Geometric sequences (with common ratio not equal to 1, 1 or 0)
show exponential growth or exponential decay
(viii)

Discuss the sum of geometric sequence

(ix) Discuss the sum of geometric infinite series

(iv)

Relation between Arithmetic mean, Multiplicative mean and Harmonic mean.

Hints: let a and b are different positive numbers

ab
2.a.b
a.b
2
ab

1. AM>GM>HM i.e
2. AM=GM=HM (If a and b are equal)
7. SETS
(i)
(ii)
(iii)
(iv)
(v)
(vi)
(vii)
(viii)
(ix)
(x)

(xi)

Define Sets with the help of real life examples


Explain how to represent the sets
Discuss cardinal number of sets
Discuss finite sets
Discuss infinite sets
Discuss Singleton sets
Discuss null sets
Discuss Universal sets with example and let students know how to
represent Universal set
Let students know Universal set on sets with examples
Discuss operation on sets
1. Union
2. Intersection
3. Difference
Discuss Theorem for sets

1. Addition Theorem of sets for two sets


2. Addition Theorem of sets three sets (most of the cases ETS test for two
sets only)
3. Addition theorem of sets for disjoint sets
Examples OG 13th Edition Chapter 5 : Q14, 192
Q82, 85, 92

Chapter 6:

SESSION-6-CLASS ASSIGNMENT (HAND OUTS-MODULE-2ALGEBRA)


GMAT/SESSION 5
Topics to be covered

Inequalities
Arithmetic Progression
Geometric Progression
In Class Problems

1. If p and q are integers, is the sum of p and q less than zero?


(1) p + 3 < q 1
(2) -2p > 2q
2. If (xy) = xy, what is the value of x + y?
(1) x = -1/2
(2) y is not equal to 0.
3.

Is x > y?
(1) x2 > y

4. Is d negative?
(1) e + d = -12

(2) x < y
(2) e d < -12

5. If -1 < x < 0, , which of the following must be true?


I.
x3 < x2
II.
x5 < 1 x
III.
x4 < x2
a. I only
b. I and II only
c. II and III only
d. I and III only
e. I, II and III
6.
a.
b.
c.
d.
e.
7.

What is the sum of the multiples of 7 from 84 to 140, inclusive?


896
963
1008
1792
2016
In a sequence of terms in which each term is three times the previous term, what is the
fourth term?
(1) The first term is 3.
(2) The second to last term is 310

8. If n is a nonzero integer, is xn < 1?


(1) x > 1

(2) n > 0
9.

If x is an integer, is 3x less than 500?


(1) 4x1 < 4x 120
(2) x2 = 36

10.If 6xy = x2y + 9y, what is the value of xy?


(1) y x = 3
(2) x3 < 0
11.A square is drawn by joining the midpoints of the sides of a given square. A third square
is drawn inside the second square in the same way and this process continues
indefinitely. If the side of the first square is 4 cm determine the sum of the areas of the
squares.
A. 32 sq. units
B. 48 sq. units
C. 64 sq. units
D. 12 sq. units
E. 120 sq. units
12.A ball is dropped from a height of 120 feet and bounces back to 3/4 th of its height
whenever it touches the ground, till it comes to rest. Find the total distance travelled by
ball.
A.
B.
C.
D.
E.

240
480
840
960
None of the above

13.If x2 2 = 1.96, y1/3 5 = 2, and x. y > 0, then what is the value of x. y?


A. 5
B. 27
C. 125
D.135
E.225
14. Patrick is cleaning his house in anticipation of the arrival of guests. He needs to
vacuum the floors, fold the laundry, and put away the dishes after the dishwasher
completes its cycle. If the dishwasher is currently running and has 55 minutes
remaining in its cycle, can Patrick complete all of the tasks before his guests arrive in
exactly 1 hour?
(1) Vacuuming the floors and folding the laundry will take Patrick 36 minutes.
(2) Putting away the dishes will take Patrick 7 minutes.
Answer key
1. B
2. C
3. C
4. C
5. E
6. C
7. A
8. C
9. C
10.B
11.A
12.C
13.D
14.B

Homework Assignment; EGE Math workbook: Chapter 2 All remaining Questions

SESSION-7 (MODULE-3-GEOMETRY)
Topics to be covered
Lines & Angles
Polygon
Triangles
1. LINES ANGLES PARALLEL AND PERPENDICULAR LINES
(i)
Define lines
(ii)
Line segment
(iii)
Intersecting Lines
(iv)
Parallel Lines
(v)
Perpendicular Lines
(vi)
Define Angles
(vii) Discuss units to measure angles ( mainly degree)
(viii) Discuss different kinds of angles
(ix)
Zero Angle
(x)
Acute Angle
(xi)
Right Angle
(xii) Obtuse Angle
(xiii) Reflex Angle
(xiv) Straight Angle
(xv) Complete Angle
(xvi) Complementary Angles
(xvii) Supplementary Angles
(xviii) Opposite Angles
(xix) Vertically Opposite Angles
(xx)
Directly opposite angles in polygon like hexagon and octagon
(xxi) Linear pair Angles
(xxii) Co-initial Angles
(xxiii) Co-terminal Angles
(xxiv) Adjacent Angles
(xxv) Different kinds of angles if two parallel lines intersected by a transversal line
(xxvi) Exterior Angles
(xxvii) Interior Angles
(xxviii)
Corresponding Angles
(xxix) Interior Alternate Angles
(xxx) Exterior Alternate Angles
2. POLYGONS
(i)
Define polygon (Closed Geometrical figure formed by three of more line segment
meeting at their end points known as polygon)
(ii)

Discuss different names of polygon

(iii)

Triangle (3 side Polygon)

(iv)

Quadrilateral (4 side Polygon)

(v)

Pentagon (5 side Polygon)

(vi)

Hexagon (6 side Polygon)

(vii)

Heptagon (7 side Polygon)

(viii)

Octagon (8 side Polygon)

(ix)

Nonagon (9 side Polygon)

(x)

Decagon (10 side Polygon)

(xi)

Discuss different kind of polygon

(xii)

In Regular polygon all sides and interior angles are same.

(xiii)

Regular polygons are always convex.

(xiv)

In Concave polygon interior angles are more than 180.

(xv)

In Convex all interior angles are less than 180.

(xvi)

Sum of Interior Angles 1800(n-2) where n is the number of sides

(xvii) Each interior angle of a regular polygon is given by:

1800 ( n 2)
n

number of sides.

(xviii) Number of diagonals in a polygon of side n is given by :

n( n 3)
2

(xix)

Sum of all exterior angles of any polygon is always 360 0

(xx)

Give complete clarity about exterior Angles of any polygon

(xxi)

Each exterior angle of a regular polygon is given by:

3600
n

where n is the

where n is the number

of sides.
3. TRIANGLES including isosceles, equilateral and 30-60-90 triangles other polygons,
congruent and similar figures, the Pythagorean theorem and angle measurement in
degrees.
(i)

Define triangle (3-sided polygon is known as triangle.)


Hints: Triangle A closed figure consisting of three line segments linked end-toend.

(ii)

Discuss terms related to triangle

(iii)

Give some Idea of Vertex (The vertex (plural: vertices) is a corner of the triangle.
Every triangle has three vertices)

(iv)

Give some Idea of Base (The base of a triangle can be any one of the three sides,
usually the one drawn at the bottom)
1.

You can pick any side you like to be the base.

2.

Commonly used as a reference side for calculating the area of the

triangle.
3.

In an isosceles triangle, the base is usually taken to be the unequal side.

(v)

One given triangle may have three different base

(vi)

Give some Idea of Altitude or height (The altitude of a triangle is the


perpendicular from the base to the opposite vertex).

(vii)

One given triangle may have three different heights

(viii)

Give some Idea of Median (The median of a triangle is a line from a vertex to the
midpoint of the opposite side. Each median divides the triangle into two smaller
triangles which have the same area)

(ix)

One given triangle may have three different medians

(x)

Give some Idea of Angle Bisectors ( A line segment which bisect given angles of
a triangle and meeting to opposite side of that angle)

(xi)

Discuss acute angle triangle (How to find Area and perimeter)

(xii)

Discuss obtuse angle triangle (How to find Area and perimeter)

(xiii)

Discuss Right Angle Triangle (How to find Area and perimeter)

(xiv)

Discuss Pythagorean theorem for right angle triangle

(xv)

Discuss trigonometry identities such as Sin, Cos and Tan for particular values like
00, 300, 450, 600 and 900

(xvi)

Discuss 30-60-90 triangles, 450-450 -900 and other polygons (triangle)

A right triangle where the angles are 30, 60, and 90.

This is one of the 'standards' triangles you should be able recognize on sight. A fact you
should commit to memory is: The sides are always in the ratio

Notice that the smallest side (1) is opposite the smallest angle (30), and the longest side (2)
is opposite the largest angle (90).
A right triangle where the angles are 45, 45, and 90

This is one of the 'standards' triangles you should be able recognize on sight. A fact you
should also commit to memory is: The sides are always in the ratio

. with the

being the

hypotenuse (longest side). This can be derived from Pythagoras' Theorem. Because the base
angles are the same (both 45) the two legs are equal and so the triangle is also isosceles.
(xvii) Explain and give complete idea about in-center (Intersecting point of angle
bisector)
(xviii) Explain and give complete idea about circum-center (Intersecting point of
perpendicular bisector of side)
(xix)

Explain and give complete idea about orthocenter (Intersecting point of heights)
In right angle triangle vertex at right angle will be orthocenter

(xx)

Explain and give complete idea about CENTROID (Intersecting point of medians).
Two-thirds of the length of each median is between the vertex and the
CENTROID, while one-third is between the CENTROID and the midpoint of the
opposite side.

(xxi)

It is very important to discuss concept of similar triangles.

1. SSS property

x
y
(If two similar triangles have sides in the ratio

x2
y2
Then their areas are in the ratio

2. SAS Property (Side and side)


3. AAA Property (All three angles are equal)
(xxii) Mid Segment of a Triangle a line segment joining the midpoints of two sides of a
triangle.
(xxiii) Properties of Mid Segment of a Triangle
1. The mid segment is always parallel to the third side of the triangle.
2. The mid segment is always half the length of the third side.
(xxiv) A triangle has three possible mid segments, depending on which pair of sides is
initially joined.
Dont forget to tell triangle inequality (let assume a, b and c are sides of a
triangle)
1. a+ b <c
2. a-b>c
(xxv) Relationship of sides to interior angles in a triangle
1.

The shortest side is always opposite the smallest interior angle

2.

The longest side is always opposite the largest interior angle

(xxvi) Discuss different structure of triangles, and their properties such as how to find
their areas and perimeters.
1. Scalene Triangle (All sides are unequal)
2. Isosceles Triangles (important structure) Isosceles triangle two sides are equal
in length. An isosceles triangle also has two angles of the same measure;
namely, the angles opposite to the two sides of the same length. For an
isosceles triangle with given length of equal sides right triangle (included
angle) has the largest area.
3. Equilateral Triangles (very important structure)
(xxvii) Discuss Geometrical Relationship
1. For a given perimeter equilateral triangle has the largest area.
2. For a given area equilateral triangle has the smallest perimeter.
3. With an equilateral triangle, the radius of the in circle is exactly half the
radius of the circum circle.
4. In a right triangle, the midpoint of the hypotenuse is equidistant from the
three polygon vertices
5. Right triangle with a given hypotenuse has the largest area when it's an
isosceles triangle.
6.

Right triangle inscribed in circle:

If M is the midpoint of the hypotenuse, BM is half of AC. One can also say
that point B is located on the circle with diameter AC. A right triangle
inscribed in a circle must have its hypotenuse as the diameter of the circle.

The reverse is also true: if the diameter of the circle is also the triangles
hypotenuse, then that triangle is a right triangle.
7. Circle inscribed in right triangle:

Note that in picture above the right angle is C.


8. Given a right triangle, draw the altitude from the right angle.

Then the triangles ABC, CHB and CHA are similar. Perpendicular to the
hypotenuse will always divide the triangle into two triangles with the same
properties as the original triangle.
Examples : OG 13th Edition Chapter 5 Q210

Chapter 6 Q56, 73, 79

SESSION-7- CLASS ASSIGNMENT (HAND OUTS-MODULE-3-GEOMETRY)

1. How many numbers do the sets S and T have in common?


(1) S is a set of 10 numbers.
(2) T is a set of 100 numbers
1

2. In the figure above, if line


(1) y = 50
(2) z = 130

is parallel to line

, what is the value of x ?

3. What is the area of triangular region ABC above?


(1) the product of BD and AC is 20
(2) x = 45

4.

In the figure above, what is the length of AC?


(1) x + y = 13
(2) xy = 36

5. In the figure above, what is the length of segment BC ?


(1) x = 90
(2)The perimeter of ABC is 24.
6. The area of the right triangle ABC is 4 times greater than the area of the right triangle KLM.
If the hypotenuse KL is 10 inches, what is the length of the hypotenuse AB?
(1) Angles ABC and KLM are each equal to 55 degrees.
(2) LM is 6 inches.

7. In the figure above, Line l and m are parallels. If v = 2w, which of the following must be
equal to q ?
(A) v + t
(B) v t
(C) t
(D) 2v
(E) s + t

8. Triangle ABC is a right angled triangle. Also AC = 25, CB = 24 and angle ADB is a right
angle. What is the length of DB?
(A) 6.72
(B) 4.8
(C) 4.5
(D)2.4
(E) 2.2

9. In the figure above AD = 4, AB = 3 and CD = 9. What is the area of triangle AEC?

(A) 22
(B) 18
(C) 9
(D)4.5
(E) 3

10.In the figure above, x > 900 and y = z + 1. If z is an integer, what is the greatest
possible value of y?
(A) 300
(B) 440
(C) 450
(D) 610
(E) 890
11.If k is an integer and 2 < k < 7, which of the following could be the value of K if there is
a triangle with sides of lengths 3, 7 and k?
(A) 3
(B) 4
(C) 5
(D) 7
(E) 9
12.If l1, l2 and l3 are lines in a plane, is l1 perpendicular to l3?
(1) l1 is perpendicular to l2.
(2) l2 is perpendicular to l3

13. In the figure above, D is a point on side AC of ABC. Is ABC is isosceles?


(1) The area of triangular region ABD is equal to the area of triangular region DBC.
(2) BDAC and AD = DC
14. If the area of triangular region RST is 25, what is the perimeter of RST?

5 2
(1) The length of one side of RST is

(2) RST is a right isosceles triangle.

15. In the figure above, line AC represents a seesaw that is touching level ground at point A. If
B is the midpoint of AC, how far above the ground is point C?
(1) x = 30

(2) Point B is 5 feet above the ground.

Answer Key
1. E
2. D
3. A
4. A
5. D
6. D
7. E
8. D
9. D
10.C
11.C
12.C
13.B
14.B
15.B
Homework Assignment:
EGE Math workbook: Chapter 3 Q1-15, 34, 36, 37, 38, 40-46

SESSION-8 (MODULE-3-GEOMETRY)
Topics to be covered
Quadrilaterals
Circles
QUADRILATERAL
(I)

Define quadrilateral (Polygon of four sides is known as quadrilateral)

(II)

Discuss the name of quadrilaterals which is tested by ETS


1. Rectangle
2. Square
3. Parallelogram
4. Rhombus
5. Trapezium

(iii) Discuss the Tips for Rectangle:


1. A 4-sided polygon where all interior angles are 90

2. Opposite sides are parallel and congruent


3. The diagonals bisect each other
4. The diagonals are congruent
5. A square is a special case of a rectangle where all four sides are the same
length
6. It is also a special case of a parallelogram but with extra limitation that the
angles are fixed at 90
7. The two diagonals are congruent (same length)
8. Each diagonal bisects the other. In other words, the point where the diagonals
intersect (cross), divides each diagonal into two equal parts
9.

Each diagonal divides the rectangle into two congruent right triangles.
Because the triangles are congruent, they have the same area, and each
triangle has half the area of the rectangle

Diagonal d l 2 b 2
10.

Where: b is the width of the rectangle; l is the length of


the rectangle.

11.The area of a rectangle is given by the formula = length X Breadth

Perimeter p 2(l b)

12.
13.A square is a special case of a rectangle where all four sides are the same
length.
14. It is also a special case of a parallelogram but with extra limitation that the
angles are fixed at 90.
(iii)

Discuss the tips of Square


1. A 4-sided regular polygon with all sides equal and all internal angles 90
2. If the diagonals of a rhombus are equal, then that rhombus must be a square
3. The diagonals of a square are (about 1.414) times the length of a side of the
square
4. A square can also be defined as a rectangle with all sides equal, or a rhombus
with all angles equal, or a parallelogram with equal diagonals that bisect the
angles

5.

If both a rectangle (right angles) and a rhombus (equal edge lengths), then it
is a square. (Rectangle (four equal angles) + Rhombus (four equal sides) =
Square)

6. A square has a larger area than any other quadrilateral with the same
perimeter
7. Like most quadrilaterals, the area is the length of one side times the
perpendicular height. So in a square this is simply: area = s 2, where s is the
length of one side.
8. The "diagonals" method. If you know the lengths of the diagonals, the area is
half the product of the diagonals. Since both diagonals are congruent (same

Area of Square

d2
2

length), this simplifies to:

, where d is the length of either

diagonal
9. Each diagonal of a square is the perpendicular bisector of the other. That is,
each cuts the other into two equal parts, and they cross and right angles
(90)

a 2
10.The length of each diagonal is

where a is the length of any one side

11.A square is both a rhombus (equal sides) and a rectangle (equal angles) and
therefore has all the properties of both these shapes, namely: The diagonals
of a square bisect each other.
12.The diagonals of a square bisect its angles.
13. The diagonals of a square are perpendicular.
14.Opposite sides of a square are both parallel and equal.
15.All four angles of a square are equal. (Each is 360/4 = 90 degrees, so every
angle of a square is a right angle.)
16. The diagonals of a square are equal
17.A square can be thought of as a special case of other quadrilaterals, for
example
a rectangle but with adjacent sides equal a parallelogram but with adjacent
sides equal and the angles all 90 a rhombus but with angles all 90
(V)

Discuss the Tips of Parallelogram


1.

A quadrilateral with two pairs of parallel sides

2.

Opposite sides of a parallelogram are equal in length

3.

Opposite angles of a parallelogram are equal in measure.

4.

Opposite sides of a parallelogram will never intersect.

5.

The diagonals of a parallelogram bisect each other.

6.

Consecutive angles are supplementary, add to 180.

7.

The area of a parallelogram is Area A = b X h, where b is the base of the


Parallelogram and h is its height.

8.

The area of a parallelogram is twice the area of a triangle created by one

of
Its diagonals
9. A parallelogram is a quadrilateral with opposite sides parallel and congruent.
It is the "parent" of some other quadrilaterals, which are obtained by adding
restrictions of various kinds
(i)

A rectangle is a parallelogram but with all angles fixed at 90

(ii)

A rhombus is a parallelogram but with all sides equal in length

(iii)

A square is a parallelogram but with all sides equal in length and all angles
fixed at 90

(V) Discuss the Tips of Rhombus


1.
A quadrilateral with all four sides equal in length
2.
A rhombus is actually just a special type of parallelogram. Recall that in a
parallelogram each pair of opposite sides is equal in length. With a rhombus, all
four sides are the same length. It therefore has all the properties of a
parallelogram
3.
The diagonals of a rhombus always bisect each other at 90.
4.
There are several ways to find the area of a rhombus. The most common is:

Area of R hom bus base height


Area of R hom bus

(VI)

d1 d 2
2

(Area in terms of diagonals)


Discuss the Tips of Trapezium
1. A quadrilateral which has at least one pair of parallel sides
2. Base -One of the parallel sides. Every trapezoid has two bases.
3. Leg - The non-parallel sides are legs. Every trapezoid has two legs.
4. Altitude - The altitude of a trapezoid is the perpendicular distance from one
base to the other. (One base may need to be extended)
5. If both legs are the same length, this is called an isosceles trapezoid, and
both base angles are the same
6. If the legs are parallel, it now has two pairs of parallel sides, and is a
parallelogram
7. Median - The median of a trapezoid is a line joining the midpoints of the two
legs
8. The median line is always parallel to the bases.
9. The length of the median is the average length of the bases
10.The median line is halfway between the bases.
11.The median divides the trapezoid into two smaller trapezoids each with half
the altitude of the original
12.Area - The usual way to calculate the area is the average base length times
altitude. The area of a trapezoid is given by

1
Area of Trapeziod h (a b)
2
(i)

Area of Trapeziod Altitude Median


or

Define circle (A line forming a closed loop, every point on which is a fixed
distance from a center point. Circle could also be defined as the set of all points
equidistant from the center)

(ii)

Discuss the terms related to circle


1. Center -a point inside the circle. All points on the circle are equidistant (same
distance) from the center point
2. Radius - the distance from the center to any point on the circle. It is half the
diameter
3. Diameter -t he distance across the circle. The length of any chord passing
through the center. It is twice the radius

4. Circumference - the distance around the circle


5. Area - strictly speaking a circle is a line, and so has no area. What is usually
meant is the area of the region enclosed by the circle
6. Chord - line segment linking any two points on a circle
7. Tangent -a line passing a circle and touching it at just one point. The tangent
line is always at the 90 degree angle (perpendicular) to the radius of a circle.
8. Secant a line that intersects a circle at two points
9. A circle is the shape with the largest area for a given length of perimeter (has
the highest area to length ratio when compared to other geometric figures
such as triangles or rectangles)
10.To form a unique circle, it needs to have 3 points which are not on the same
line
11.Chord- a line that links two points on a circle or curve
12.Discuss the formula to find circumference of semi-circle
13.Discuss the formula to find area of semi-circle
14.Discuss the formula to find perimeter of semi-circle
(VII) Discuss the properties and theorem of circles
1.

The angle inscribed in a semicircle is always 90

2. Any diameter of a circle subtends a right angle to any point on the circle. No matter
where the point is, the triangle formed with diameter is always a right triangle
3. In a circle, a radius perpendicular to a chord bisects the chord. Converse: In a circle,
a radius that bisects a chord is perpendicular to the chord, or in a circle, the
perpendicular bisector of a chord passes through the center of the circle
4. Given two points A and C, lines from them to a third point B form the inscribed angle
ABC. Notice that the inscribed angle is constant. It only depends on the position of
A and C
5. A central angle is an angle AOC with endpoints A and C located on a circle's
circumference and vertex O located at the circle's center. A central angle in a circle
determines an arc AC
6. The Central Angle Theorem states that the measure of inscribed angle is always half
the measure of the central angle
7. An inscribed angle is exactly half the corresponding central angle. Hence, all
inscribed angles that subtend the same arc are equal. Angles inscribed on the arc
are supplementary. In particular, every inscribed angle that subtends a diameter is a
right angle (since the central angle is 180 degrees)
(VII)

Arcs and Sectors (A portion of the circumference of a circle)

(VIII) Major and Minor Arcs Given two points on a circle, the minor arc is the
shortest arc linking them
(IX)

Discuss the formulae to find Arc Length

(X)

Discuss the formulae to find area of Sector

(XI)

Power of a Point Theorem (Given circle O, point P not on the circle, and a
line through P intersecting the circle in two points. The product of the
length from P to the first point of intersection and the length from P to the

second point of intersection is constant for any choice of a line through P


that intersects the circle. This constant is called the "power of point P")
Discuss the formulae for different cases
1. If P is outside the circle: (intersecting secants)
2. If P is inside the circle: (Intersecting chords)

Examples: OG 13th Edition


Chapter 5: Q36, 165, 213
Chapter 6 : Q119, 122, 145, 152

SESSION 8-CLASS ASSIGNMENT (HAND OUTS-MODULE-3-GEOMETRY)


1. In the figure above, the length of CD to ED is 4:3, and the length of each side of
quadrilateral ABCE is same but a whole number. Which of the following could be the
area of quadrilateral ABCD?
(A) 532
(B) 416
(C) 102
(D) 28
(E) 22

2. In the figure above, regular pentagon ABCDE is divided into three non overlapping
triangles. Which of the following must be true about the three triangles?
(A) They have equal areas.
(B) They have equal perimeters.
(C) They are similar.
(D) They are isosceles.
(E) They each have at least one angle of measure 60.

3. In the figure above, the radius of the circle with center R is twice the radius of the
circle with center P. What is the radius of the circle with center R?
(A) 5
(B) 6
(C) 7
(D) 8
(E) 10

4. In the figure above, the circle with center O is inscribed in square WXYZ. What is the
area of the shaded portion of the figure?
(A) 8 2
(B) 8
(C) 4 2
(D) 2 (-2)
(E) 4 3

5. Point O is the center of both circles in the figure above. If the circumference of the
large circle is 36 and the radius of the small circle is at half of the radius of the large
circle, what could be the length of the darkened arc?
(A) 10
(B) 8
(C) 5
(D) 4
(E) 2

6. A circle (not drawn) passes through point A in the figure above. Which of the
following cannot be the total number of points of intersection of this circle and
ABC?
(A)
1
(B)
2
(C)
3
(D)
5
(E)
6
7. What is the perimeter of rectangle R?
(1) R is a square.
(2) The area of R is 36.

8. The figure above shows the circular cross section of a concrete water pipe. If the inside
radius of the pipe is r feet and the outside radius of the pipe is t feet, what is the value of
r?
(1) The ratio of t - r to r is 0.15 and t - r is equal to 0.3 foot.
(2) The area of the concrete in the cross section is 1.29 square feet.

9. The figure above shows the present position on a radar screen of a sweeping beam that is
rotating at a constant rate in a clockwise direction. In which of the four quadrants will the
beam lie 30 seconds from now?
(1) In each 30-second period, the beam sweeps through 3690
(2) r = 40
10. What is the area of rectangular region R?
(1) Each diagonal of R has length 5.
(2) The perimeter of R is 14.

11. The circular base of an above-ground swimming pool lies in a level yard and just
touches two straight sides of a fence at points A and B, as shown in the figure above. Point
C is on the ground where the two sides of the fence meet. How far from the center of the
pool's base is point A?
(1) The base has area 250 square feet.
(2) The center of the base is 20 feet from point C
12. The figure above shows the shape of a flower bed. If arc QR is a semicircle and PQRS is
a rectangle with QR > RS, what is the perimeter of the flower bed?
(1) The perimeter of rectangle PQRS is 28 feet.
(2) Each diagonal of rectangle PQRS is 10 feet long.
13 Is quadrilateral RSTV a rectangle?
(1) The measure of angle RST is 90 degrees
(2) The measure of angle TVR is 90 degrees
14

Is the perimeter of rectangle R greater than 28?

(1)The area of rectangle R is 50.


(2) The diagonal of rectangle R is 10.

15.Quadrilateral RSTU shown above is a site plan for a parking lot in which side RU is
parallel to side ST and RU is longer than ST. What is the area of the parking lot ?
(1) RU = 80 meters
20 10
(2) TU=
meters

Answer key
1. B
2. D
3. B
4. D
5. D
6. E
7. C
8. A
9. A
10.C
11.A
12.C
13.E
14.A
15.D
Homework Assignment:
OG 12th Edition
Chapter 5: 4, 16, 18, 33, 48, 53, 62, 102, 113, 134, 145, 147, 152, 160, 177, 189, 197, 209,
212
Chapter 6 : 18, 20, 29, 34, 42, 47, 56, 74, 109, 114, 117, 122, 132, 135, 140, 144, 148, 149,
157, 160, 173

SESSION-9 (MODULE-3-GEOMETRY)
Topics to be covered
Coordinate Geometry
Solid Geometry
1.

CO-ORDINATE GEOMETRY
(i)
Discuss what the use of using coordinate geometry?
(ii)
Discuss the definition of coordinate geometry (Coordinate geometry, or Cartesian
geometry, is the study of geometry using a coordinate system and the principles
of algebra and analysis)
(iii)
Discuss coordinate plane (The coordinate plane is a two-dimensional surface on
which we can plot points, lines and curves. It has two scales, called the x-axis
and y-axis, at right angles to each other)
(iv)
Discuss X axis (The horizontal scale is called the x-axis and is usually drawn with
the zero point in the middle. Values to the right are positive and those to the left
are negative)
(v)
Discuss Y axis (The vertical scale is called the y-axis and is also usually drawn
with the zero point in the middle. Values above the origin are positive and those
below are negative)
(vi)
Discuss Origin (The point where the two axes cross (at zero on both scales) is
called the origin)
(vii) Discuss Quadrants (When the origin is in the center of the plane, they divide it
into four areas called quadrants)
(viii) Discuss Point (The coordinates are written as an "ordered pair". The letter P is
simply the name of the point and is used to distinguish it from others)
(ix)
Discuss the formulae for distance between two points
(x)
Discuss Vertical and horizontal lines (If the line segment is exactly vertical or
horizontal, the formula above will still work fine, but there is an easier way. For a

(xi)
(xii)

horizontal line, its length is the difference between the x-coordinates. For a
vertical line its length is the difference between the y-coordinates)
Discuss Distance between the point A (x, y) and the origin
Discuss Midpoint of a Line Segment (A line segment on the coordinate plane is
defined by two endpoints whose coordinates are known. The midpoint of this line
is exactly halfway between these endpoints and its location can be found using
the Midpoint Theorem, which states:
1.
The x-coordinate of the midpoint is the average of the x-coordinates of the
two endpoints.
2.
Likewise, the y-coordinate is the average of the y-coordinates of the
endpoints

(xiii)
(xiv)

Let students know, Lines and its properties is main concept tested by ETS.
Discuss Lines (Every straight line in the plane can represent by a first degree
equation with two variables)
(xv) Discuss General form of line (t he general form of the equation of a straight line
is ax + by + c = 0)
(xvi) Discuss slope intercept form line of line which is y = m.x + c. Where m is slope
and c is intercept on y axis.
(xvii) Discuss point form of line and its slope. Slope in terms of angle subtend between
x-axis and line in anti-clockwise direction.

Equation of line passes by po int s ( x1 , y1 ) and ( x 2 , y 2 ) is...


y y1
Here

y2 y1
( x x1 )
x2 x1

y2 y1
m tan Slope
x2 x1

(xviii) Discuss Point slope form of line

Equation of line passes by po int s ( x1 , y1 ) and having slope m


y y1 m ( x x1 )
(xix)

Discuss Intercept form of line

Equation of line having int ercept a and b on x axis and y axis respectively
x y
1
a b

(xx)
(xxi)

Discuss slope direction (the slope of a line can be positive, negative, zero or
undefined)

Discuss Positive slope (y increases as x increases, so the line slopes upwards to


the right. The slope will be a positive number)
(xxii) Negative slope (y decreases as x increases, so the line slopes downwards to the
right. The slope will be a negative number)
(xxiii) Discuss Zero slope (y does not change as x increases, so the line in exactly
horizontal. The slope of any horizontal line is always zero. The line below goes
neither up nor down as x increases, so its slope is zero)
(xxiv) Discuss undefined slope (When the line is exactly vertical, it does not have a
defined slope. The two x coordinates are the same, so the difference is zero is
the situation of not defined slope)
(xxv) Give closer look for slope and quadrants
1.
If the slope of a line is negative, the line WILL intersect quadrants II and IV. X and Y
intersects of the line with negative slope have the same sign. Therefore if X and Y intersects
are positive, the line intersects quadrant I; if negative, quadrant III
2.
If the slope of line is positive, line WILL intersect quadrants I and III. Y and X intersects
of the line with positive slope have opposite signs. Therefore if X intersect is negative, line
intersects the quadrant II too, if positive quadrant IV

3. Every line (but the one crosses origin OR parallel to X or Y axis OR X and Y axis themselves)
crosses three quadrants. Only the line which crosses origin OR is parallel to either of axis
crosses only two quadrants
4. If a line is horizontal it has a slope of 0, is parallel to X-axis and crosses quadrant I and II if
the Y intersect is positive OR quadrants III and IV, if the Y intersect is negative. Equation of
such line is y=b, where b is y intersects.
5.
If a line is vertical, the slope is not defined, line is parallel to Y-axis and crosses quadrant
I and IV, if the X intersect is positive and quadrant II and III, if the X intersect is negative.
Equation of such line is x = a, where a is x-intercept.
6. If the slope is 1 the angle formed by the line is 450degrees
(xxvi) Discuss how to find the equation line which parallel to a given line
(xxvii) Discuss how to find the equation line which perpendicular to a given line
(xxviii)
Discuss formula to find the distance between a line and a point

Dis tan ce between line ax by c 0 po int ( x1 , y1 ) is...

ax1 by1 c
a 2 b2

(xxix) Discuss the formula how to find angle between two intersecting lines

tan

m2 m1
1 m1m2

m2 m1
(xxx) Discuss the condition if lines are parallel

m1m2 1

(xxxi) Discuss the condition if lines are perpendicular


(xxxii) Briefly discuss the circle on a plane (In an x-y Cartesian coordinate system, the
circle with center (a, b) and radius r is the set of all points (x, y) such that :)

( x a ) 2 ( y b) 2 r 2
2.

SOLID GEOMETRY
(I)

Define and explain solids (Solid geometry is concerned with threedimensional shapes. Some examples of three-dimensional shapes are
cubes, rectangular solids, prisms, cylinders, spheres, cones and pyramids.
We will look at the volume formulas and surface area formulas of the
solids)

(ii)

Kind of solid this is tested by ETS (There are two types of solid one is plane
surfaces other is curved surfaces?)

(iii)

Cubes: A cube is a three-dimensional figure with six matching square

sides.
If s is the length of one of its sides
The area of each side of a cube is s2. Since a cube has six square-shape
sides, its total surface area is 6 times s2.
Surface area of a cube = 6s2
Lateral Surface Area of cube = 4s2
Volume of the cube = s3

s 3
(iv)
(v)

Longest Diagonal
Discuss complete information about Rectangular Solids or Cuboids
A rectangular solid is also called a rectangular prism or cuboids. (In a
rectangular solid, the length l, width w and height h may be of
different lengths)

Volume of rectangular solid = lwh


Total area of top and bottom surfaces is lw + lw = 2lw
Total area of front and back surfaces is lh + lh = 2lh
Total area of the two side surfaces is wh + wh = 2wh
Surface area of rectangular solid = 2lw + 2lh + 2wh = 2(lw + lh + wh)
First let students know what lateral surface area is and then give its formula
Lateral Surface Area = 2(lh + wh)

l 2 w2 h 2
Longest diagonal d=
(i)
(ii)
(iii)
(iv)
(v)

Discuss about Cylinders and its properties (A cylinder is a solid with two
congruent circles joined by a curved surface)
If in the radius of the circular base is r and the height is h. The volume
of the cylinder is the area of the base height.
Surface area = 2 area of circle + area of rectangle. Surface area of
cylinder = 2r2 + 2rh = 2r (r + h)
First let students know what is lateral surface area and then give its
formula Lateral Surface Area = 2rh
Discuss about Sphere and its properties (A sphere is a solid with all its
points the same distance from the center)

Volume and Surface area of shpere whose radius is r


4
Volume V r 3
3
Surface Area S 4 r 2

(vi)
(vii)

Give some idea about Semi or Hemi Sphere


Discuss about semi Sphere and its properties (A sphere is a solid with
all its points the same distance from the center)

Volume and Surface area of shpere whose radius is r


2
Volume V r 3
3
Surface Area S 2 r 2
Total Surface Area S 3 r 2
(viii)
(ix)

Discuss about right circular cone (A circular cone has a circular base,
which is connected by a curved surface to its vertex. A cone is called a
right circular cone, if the line from the vertex of the cone to the center
of its base is perpendicular to the base)

(x)

Give brief idea about Pyramid in two minute (A pyramid is a solid with a
polygon base and connected by triangular faces to its vertex. A
pyramid is a regular pyramid if its base is a regular polygon and the
triangular faces are all congruent isosceles triangles)

Volume of pyramid V
(xi)
(xii)

1
Area of base height
3

Volume, Lateral and Surface area of right circular cone whose


radius and height are r and h respectively
1
Volume V r 2 h
3
Lateral Surface Area S rl
Total Surface Area S rl 2 r 2

GIVE SOME TIPS FOR GEOMETRICAL RELATIONSHIP


(i)

From all 3-D figure with given volume, sphere is one of which has least surface
area

(ii)

From all 3-D figure with given surface area, sphere is one of which has greatest
volume

(iii)

Largest cuboids inscribed in a sphere is cube

(iv)

Radius of sphere inscribed in a cube is half of side of cube

(v)

Side of cube inscribed in a sphere is diameter of sphere

Examples: OG 13th Edition


Chapter 5 : Q7, 28, 202, 211
Chapter 6 : Q 30, 35, 155

SESSION-9-CLASS ASSIGNMENT (HAND OUTS-MODULE-3-GEOMETRY)

Q
S
O
T

1. A line in a coordinate system is graphed by y = -0.5x + 2. Which of the following


equations will yield a graph such that the area of triangle SOT will be half the size of
triangle QOR?
I. y = -0.25x + 1
II. y = -0.25x + 2
III. y = -x + 2
(A) I only
(B) II only
(C) III only
(D) I and II only
(E) I and III only
Y
A
x
O

2. In the rectangular coordinate system above, if OA < AB.Is the area of region OAB
greater than 48?
(1) The coordinates of point A are (6, 8).
(2) The coordinates of point B are (13,0).
3. In the rectangular coordinate system, which quadrant, if any, contains no point (x, y)
that satisfies the inequality 2x 3y -6?
(A) None
(B) I
(C) II
(D) III
(E) IV
4.
E (4, 4)

O
D

F (4, -1)

In the rectangular coordinate system above, if the area of right triangle DEF is 15, then
which of the following are the coordinates of point D?

A.
B.
C.
D.
E.

(4, 1)
(2, 1)
(2, 4)
(1, 1)
It cannot be determined from the information given.

5.

y
o

On the coordinate axes shown above, the graph of y = 4x + 20 would cross the x-axis
at the point where
A.
B.
C.
D.
E.

x
x
x
x
x

=
=
=
=
=

5 and y = 0
0 and y = 5
0 and y = 5
0 and y = 20
5 and y = 0

6. Points (a, b) and (c, d) are in quadrants I and III, respectively. If abcd 0, then the
point (bd, bc) must be in which quadrant?
A. I
B. II
C. III
D. IV
E. It cannot be determined from the information given.

7. In the figure above, if line k has slope of -1, what is the y-intercept of k?
A. 4
B. 5
C. 6
D. 7
E. 8
8. For any triangle T in the xycoordinate plan, the center of T is defined to be the point
whose xcoordinate is the average (arithmetic mean) of the xcoordinates of the
vertices of T and whose ycoordinate is the average of the ycoordinates of the vertices
of T. If a certain triangle has vertices at the points (0,0) and (6,0) and center at the
point (3,2), what are the coordinates of the remaining vertex?
A.
B.
C.
D.
E.

(3,4)
(3,6)
(4,9)
(6,4)
(9,6)

9. In the xyplane, at what two points does the graph of y = (x+a) (x+b) intersect the
xaxis?
(1) a + b = 1
(2) The graph intersects the yaxis at (0, 6).
10. Circle C and line k lie in the xyplane. if circle C is centered at the origin and has
radius 1, does line k
Intersect circle C?
(1) The xintercept of line k is greater than 1
(2) The slope of line k is 1/10.
11. In the xy plane, what is the yintercept of the line l?
(1) The slope of the line l is 3 times its y intercept.
(2) The xintercept of line l is 1/3
12. Line l is defined by the equation y 5x = 4 and line w is defined by the equation
10y + 2x + 20 = 0. If line k does not intersect line l, what is the degree measure of the
angle formed by line k and line w?
A. 0
B. 30
C. 60
D. 90
E. It cannot be determined from the information given.
13. Point K = (A, 0), Point G = (2A + 4, (2A+ 9)). Is the distance between point K and
G prime?
(1) A2 5A 6 = 0
(2) A > 2

Answer Keys
1. E
2. A
3. E
4. B
5. A
6. D
7. B
8. .
9. .
10..
11..
12..
13..
14..
15..
Homework Assignment:
EGE Math Workbook
Chapter 3 (All remaining Questions)
Chapter 5 - Geometry

SESSION-10 (MODULE-4-COMBINATORICS)
Topics to be covered
Permutations

Combinations
1. COMBINATORY
(i)
Explain the meaning of combinatory (Combinatory is branch of mathematics that
deals with collections of objects that satisfy specified criteria)
(ii)
What comes under this concept
1. Counting theorems
(i)
Multiplication Theorem of counting
(ii)
Addition Theorem of counting
2. Permutations
3. Combinations
(I)
Discuss Factorials
(II)
Let students know 1! And 0! Both has same value as 1
(III)
Let students know factorial of any negative integers is always not defined. Give
little explanation with one or two examples
(IV)
Discuss Multiplication Theorem of counting with at least 5 examples
Hints: If any task A can be done in m ways, task B can be done in n ways and
task C can be done if and only if task A and task B both have already done then
total number of ways of doing task C can be obtained by using Multiplication
Theorem Counting
Total number of ways of doing task C = m x n ways.
(V)

Discuss Addition Theorem of counting with at least 3 examples


Hints: If any task A can be done in m ways, task B can be done in n ways and
task C can be done if one of the task A or task B have already done then total
number of ways of doing task C can be obtained by using Addition Theorem
Counting.
Total number of ways of doing task C = m + n ways

(VI)

Define permutations and combination discuss how permutations differ from


combinations with examples
Hints1: Total number of arrangement of r things from n different things if

nr
repetitions of things are allowed =
Hints2: Total number of arrangement of r things from n different things if
repetitions of things are not allowed

n!
n (n 1)(n 2)....( n r )!
np

n (n 1)( n 2)....( n r 1)
r ( n r )!
( n r )!

(VII)

Discuss restricted permutations


1. Total number of arrangement of n different things in which k particular

r !.(n r 1)!

things always Comes together =


2. Total number of arrangement of n different things in which k particular

n ! r !.(n r 1)!

things never Comes together =


(VIII) Discuss in details with examples about repeated permutations
Hints: Total number of arrangement of n different things in which p things, q

n!
p !.q !.r !
things and r things are identical=
(IX)

Give some solved example in class on the concept of restricted repeated


permutations.

(X)

Give complete clarity about circular permutations. Explain how circular


permutation and linear permutations are correlated and what the main difference
is.
Hints: there are two cases of circular-permutations:1.
If clockwise and anti clock-wise orders are different, and then total number
of circular-permutations is given by (n-1)!
2.
If clock-wise and anti-clock-wise orders are taken as not different, then

(XI)

total number of circular-permutations is given by


Discuss the combinations with examples

(XII) Give relative between combination and permutation (


(XIII) Give some standard results to remember

(n 1)!
2

n p nc r!
r
r

n!
(i ) nC
1
0
0!( n 0)!
n!
(ii ) nC
n
1
1!( n 1)!
n!
n( n 1)
(iii ) nC 2

2!( n 2)!
2
n!
n( n 1)( n 2)
(iv ) nC

3
3!(n 3)!
6
(VIV) Discuss application of combinations
App-1 Total number of selection of r different things from n different in which k

(n k )

C( r k )

particular things always occurs where k


r=
Take some examples to give more clarity on this application
App-2 Total number of selection of r different things from n different in which k

(n k )

Cr

particular things never occurs =


Take some examples to give more clarity on this application
App-3 Number of ways of selecting zero or more things from n different things
= 2n

nC nC nC 2 nC ..........nC 2n
0
1
n
3
Take some examples to give more clarity on this application
App-4 Number of ways of selecting at least one thing from n different things

nC nC 2 nC ..........nC 2 n 1
1
n
3
Take some examples to give more clarity on this application
App-5 Number of ways of selecting zero or more things from n identical things = n+1

App-6 Number of ways of selecting at least one things from n identical things = n

nC nC 2 nC ..........nC 1 1 1 ......(n )times n


1
n
3

Take some examples to give more clarity on this application


App-7 Total number of ways of dividing m + n different things into two groups of m

(m n)

Cm

(m n)

Cn

( m n )!
m !.n !

things and n things respectively =


Take some examples to give more clarity on this application
App-8 Total number of ways of dividing 2 n different things into two groups n things

(2n )!
2!.n !.n !

each
Take some examples to give more clarity on this application
App-9 Total number of ways of dividing m + n + p different things into three groups of

( m n p )!
m !.n !. p!

m things, n things and p things each respectively =


Take some examples to give more clarity on this application
App-10 Total number of ways of dividing 3ndifferent things into three groups n things

each

(3n )!
3!.n !.n !.n !

SESSION-10-CLASS ASSIGNMENT (HAND OUTS-MODULE-4-(Permutations &


Combinations)

1. A code to a certain lock consists of digits that cannot be repeated. If the code is
known to consist of at least 8 digits and it takes 12 seconds to try one combination,
what is the amount of time, in minutes, necessary to guarantee access to the lock?

(A)

5
2

8!
5

(B)

8!
2

(C)

8!

(D)

10!
2

(E)

10!

2. Melons clothing store uses a bar-code system to identify every item. Each item is
marked by a combination of 2 letters followed by 3 digits. Additionally, the three-digit
number must be even for male products and odd for female products. If all apparel
products start with the letter combination AP, how many male apparel items can be
identified with the bar code?
(A)
200
(B) 405
(C)
500
(D)
729
(E)
1000
3. How many two-element subsets of {1, 2, 3, 4} are there that do not contain the pair
of elements 2 and 4?
(A)
One
(B) Two
(C) Four
(D)
Five
(E) Six
4. SHERLEY needs to choose 3 flowers for her sister from a group of 10 roses, 6 of
which are red and 4 of which are white. What is the ratio of the number of choices
Sherley has to select only red roses to the number of choices she has to select only
white roses?
(A) 30:1
(B)
10:1

(C)
5:1
(D)
3:2
(E)
1:5
5. If 6 fair coins are tossed, how many different coin sequences will have exactly 3 tails,
if all tails have to occur in a row?
(A)
4
(B)
8
(C)
16
(D)
20
(E)
24
6. In how many ways can a teacher write an answer key for a mini-quiz that contains 3
true-false questions followed by 2 multiple-choice questions with 4 answer choices
each, if the correct answers to all true-false questions cannot be the same?
(A)
62
(B)
64
(C)
96
(D)
126
(E)
128

7. How many parallelograms are formed when a set of five parallel lines intersect
another set of four Parallel lines?
(A) 120
(B) 60
(C) 36
(D) 20
(E) 10
8. There are 5 students and 3 teachers. In how many ways can a team of 5 be formed
so that there is atleast one teacher but not more than two teachers in the team?
(A) 450
(B) 180
(C) 60
(D) 45
(E) 30
9. Find the sum of all four digit numbers that can be formed using the digits 0,1,2,3
without repetition?
(A) 39996
(B) 38664
(C) 36664
(D) 19332
(E) 18332
10. Harry receives guidelines from 12 customers and uses color coding to identify each
customer. If either a single color or a pair of two different colors is chosen to represent
each customer and if each customer is uniquely represented by that choice of one or
two colors, what is the minimum number of colors needed for the coding? (Assume that
the order of the colors in a pair does not matter.)
(A) 4
(B) 5
(C) 6
(D) 12
(E) 24

11. Right triangle ABC is to be constructed in the xy -plane so that the right angle is at
A and AC is parallel to the x-axis. The x and y coordinates of A, B and C are to be
integers that satisfy the inequalities 6 x 5 and 8 y 18. How many different
triangles with these properties could be constructed?
(A) 110
(B) 1,100
(C) 9,900
(D) 10,000
(E) 12,100
12. (A) How many different numbers between 100 and 1000 can be formed using the
digits 0,1,2,3,4,5,6 without repetition?
(A) 120
(B) 150
(C) 180
(D) 720
(E) 5040

12.(B) How many of the above numbers are divisible by 5?


(A) 36
(B) 50
(C) 55
(D) 60
(E) 1008
13. George is setting up an aquarium and must choose 4 of 6 different fish and 2 of 3
different plants. How many different combinations of fish and plants can George
choose? Ans (D)
(A) 8
(B) 12
(C) 18
(D) 45
(E) 90
14. Six students compete in a table tennis tournament. Each student plays each of the
other students four times. What is the total number of games played in the
tournament? ANS (D)
(A) 15
(B) 30
(C) 45
(D) 60
(E) 120
15. Six students compete in a table tennis tournament. Each student plays each of the
other students four times. What is the total number of games played in the
tournament?
(A) 6
(B) 8
(C) 9
(D) 12
(E) All of the above

Answer Key
1. D
2. C
3. D
4. C
5. A
6. C
7. B
8. D
9. B
10.B
11.C
12.(I) C
12(II) C
13.D
14.D
15.E
Homework Assignment
EGE Math workbook
Chapter 4 : Q26-50

SESSION-11 (MODULE-4-COMBINATORICS)
Topic to be covered
Probability
1. PROBABILITY
(i)
Define probability (The probability of an event occurring is the likelihood of it
happening expressed in mathematical terms. The likelihood of an event
occurring is the number of ways that particular event can occur divided by the
number of ways any possible outcome can occur.)
(ii)
Discuss what is the exact meaning of probability (briefly)
(iii)
What is the use and importance of probability in real life (briefly)
(iv)
Before begin real probability concepts lets discuss the concepts related to
probability
(v)

Discuss what is experiment?


Hints: Happening of anything if result is uncertain, known as experiment. For
example, tossing a coin, tossing a dice, picking a number of cards from a well
shuffled pack of 52 cards etc

(vi)

Discuss what is Outcome?


Hints: Individual result of an experiment is known as outcome
For example, getting even number on a toss of a single dice

(vii)

Discuss what is Sample Space?


Hints: Total number of all possible outcomes is known as sample space. For
example, Sample space is 2 if a single coin tossed. Sample space is 4 if two coins
are tossed. Sample space is 8 if three coins are tossed. Sample space is 2 n if n
coins are tossed.

(viii)

Discuss what is Event?


Hints: An outcome to a random occurrence (one or more possible outcome of an
experiment). For example, the following are events: drawing a black card from a
52-card deck, a coin landing on heads, rolling an even number on a 6-sided die.

(ix)

Discuss what is Simple Event?


Hints: An event with single outcome. For example, getting head in a single
thrown of a coin

(x)

Discuss what is Certain Event?

Hints: If an event is certainly is going to happen is known as certain event. For


example, Probability of getting a number less than equal to six on a tossed of a
dice
(xi)

Discuss what is Impossible Event?


Hints: If an event is certainly is not going to happen is known as impossible
event.
For example, Probability of getting a number greater than six on a tossed of a
dice

(xii)

Discuss what is Intersection of Events?


Hints: P(AB) - When two events are fulfilled simultaneously. For example, the
intersection of the events "rolling an even number" and "rolling a number less
than three" is rolling a 2 since rolling a 2 fulfills both events (i.e., 2 is both even
and less than 3).

(xiii)

Discuss what is Union of Events?


Hints: P(AB) - When either of two events is fulfilled. For example, the union of
the events "rolling an even number" and "rolling a number less than three" is
rolling a 2, 4, 6 (an even number) or rolling a 1, 2 (a number less than three).

(xiv)

Discuss what is Dependent Event?


Hints: An event whose probability of occurring is influenced by other event or
events
For example, the probability of drawing a red card from a pack of 52-card deck
after you draw another card from the 52-card deck without replacing this other
card is a dependent event. The probability of the second card you draw being red
depends on what card was drawn the first time.

(xv)

Discuss what is Independent Events?


Hints: Two events are independent if the occurrence of one event does not affect
the probability of the occurrence of the other.
For example, the probability of flipping a coin twice and the coin landing on
heads the second time is not affected by (i.e., is independent of) whether the
first coin flip turned up heads or tails.

(xvi)

Discuss what is Mutually Exclusive Events?


Hints: Two events are mutually exclusive if they cannot occur together.
For example, the events "rolling an even number" and "rolling an odd number"
are mutually exclusive since by definition rolling an even number means you
cannot roll an odd number.

(xvii) Discuss what is Complement of an Event?


Hints: The event that is composed of all the outcomes those are not in another
event. For example, the complement of flipping a coin and it landing on heads is
flipping a coin and it landing on tails. The complement of rolling an even number
is rolling an odd number. The complement of drawing a heart is drawing either a
spade, club, or diamond.
(xviii) Give little clarity about graphical representations of union of events, intersection
of events, and difference of events and also complement of an event. Let
students understand this concept is somehow similar to sets concepts.
(xix)

Discuss basic theorem and way to find probability of a given event.


Hints: Probability is a measure of the expectation that an event will occur or a
statement is true. Probabilities are given a value between 0 (will not occur) and 1

(will occur).The higher the probability of an event, the more certain we are that
the event will occur.
(xx)

Discuss addition theorem of probability (with examples)

P ( A B ) P ( A) P( B ) P( A B )

(xxi) Discuss addition theorem of probability for independent events


(xxii) Discuss addition theorem of probability for mutually exclusive events
(xxiii) Discuss addition theorem of probability for non-mutually exclusive events.
(xxiv) Discuss multiplication theorem of probability for independent events (two or
more)
(xxv) Discuss multiplication theorem of probability for two independent events and
both events are being success.

P( A B ) P ( A).P( B )

(xxvi) Discuss multiplication theorem of probability for two independent events and
both events are not being success.

P( A B ) P ( A). P( B )
(xxvii) Discuss multiplication theorem of probability for two independent events and
neither both event success and both events failure.

P ( A B ) P ( A).P( B )
P ( A B ) P ( A).P( B )
(xxviii)

Discuss De-Morgans Law of probability

P( A B ) P( A B )
Explain what the use of using DEMORGANNS of probability and let students
know even without using this law we can sole the probability problems. Give
some example for this
(xxix) Discuss ratio of odds favor of an event

Odds Favor of an event

p
q

(xxx) Discuss ratio of odds favor of an event

Odds against of an event

q
p

(xxxi) Discuss Probability distributions (Brief Idea)


Hints: In probability and statistics, a probability distribution assigns a probability
to each of the possible outcomes of a random experiment
Examples: OG 13th Edition
Chapter 5: Q68
Chapter 6: Q125

SESSION-11-CLASS ASSIGNMENT (HAND OUTS-MODULE-4PROBABILITY)


1. The probability of pulling a black ball out of a glass jar is 1/X. The probability of pulling
a black ball out of a glass jar and breaking the jar is 1/Y. What is the probability of
breaking the jar?
A. 1/(XY).
B. X/Y.
C. Y/X.
D. 1/(X+Y).
E. 1/(X-Y).
2. In jar A there are 3 white balls and 2 green ones, in jar B there is one white ball and
three green ones. A jar is randomly picked, what is the probability of picking up a white
ball out of jar A?
A. 2/5.
B. 3/5.
C. 3/10.
D. 3/4
E. 2/3.
3. Out of a box that contains 4 black and 6 white mice, three are randomly chosen. What
is the probability that all three will be black?
A. 8/125.
B. 1/30.
C. 2/5.
D. 1/720.
E. 3/10.

4. Danny, Doris and Dolly flipped a coin 5 times and each time the coin landed on
heads. Dolly bet that on the sixth time the coin will land on tails, what is the
probability that shes right?
A. 1.
B. .
C. .
D. .
E. 1/3.
5.
In a box there are A green balls, 3A + 6 red balls and 2 yellow ones. If there are
no other colors, what is the probability of taking out a green or a yellow ball?
A. 1/5.
B. 1/2.
C. 1/3.
D. 1/4.
E. 2/3.
6. In a jar there are 3 red balls and 2 blue balls. What is the probability of drawing at least
one red ball when drawing two consecutive balls randomly?
A. 9/10
B. 16/20
C. 2/5
D. 3/5
E.
7.
In Rwanda, the chance for rain on any given day is 50%. What is the probability that it
rains on 4 out of 7
consecutive days in Rwanda?
A. 4/7
B. 3/7
C. 35/128
D. 4/28
E. 28/135

8. A Four digit safe code does not contain the digits 1 and 4 at all. What is the probability
that it has at least one even digit?
A.
B.
C.
D. 15/16
E. 1/16
9. John wrote a phone number on a note that was later lost. John can remember that the
number had 7 digits, the digit 1 appeared in the last three places and 0 did not appear
at all. What is the probability that the phone number contains at least two prime digits?
A. 15/16
B. 11/16
C. 11/12
D.
E. 5/8
10.How many diagonals does a polygon with 21 sides have, if one of its vertices does not
connect to any diagonal?
A. 21
B. 170
C. 340
D. 357
E. 420
11.
A drawer holds 4 red hats and 4 blue hats. What is the probability of getting
exactly three red hats or exactly three blue hats when taking out 4 hats randomly out
of the drawer and returning each hat before taking out the next one?
A. 1/8
B.

C.
D. 3/8
E. 7/12
12.
In a department store prize box, 40% of the notes give the winner a dreamy
vacation; the other notes are blank. What is the approximate probability that 3 out of 5
people that draw the notes one after the other, and immediately return their note into
the box get a dreamy vacation?
A. 0.12
B. 0.23
C. 0.35
D. 0.45
E. 0.65
13.
The probability of having a girl is identical to the probability of having a boy. In a
family with three children, what is the probability that all the children are of the same
gender?
A. 1/8
B. 1/6
C. 1/3.
D. 1/5.
E.
14.
One person won the lottery this week, what is the probability that it was a woman
over the age of 40?
(1) 55% of all the lottery participants are male.
(2) 60% of all the lottery participants are over the age of 40.
15.
If a kid is chosen randomly from his class, what is the probability that he would
have blue eyes?
(1) The class is in Denmark, where 95% of the population has blue eyes.
(2) 5% of the class has brownish eyes.
16.
In a bulb factory there are different kinds of bulbs, what is the probability that a
bulb chosen randomly is a halogen?
(1) There are three times as many halogens than any other bulb in the factory.
(2) The ratio between the halogen to all the other bulbs is 2 to 7.

Answer Key
1. B
2. C
3. B
4. B
5. D
6. A
7. E
8. D
9. B
10. B
11. C
12. B
13. E
14. E
15. E
16. B
Homework Assignment: EGE Math workbook Chapter 4 Q1-25

SESSION-12 (MODULE-4)
Topics to be covered
Statistics
Data Interpretation
1. STATISTICS
(I)
Define statistics with examples
Hint: statistics is study about data or Statistics is the study of the collection,
organization, analysis, interpretation, and presentation of data. It deals with all
aspects of this, including the planning of data collection in terms of the design of
surveys and experiments.
(II)
Discuss briefly why to use statistics
Hints: Statistics are useful for huge data base analysis. The concept of
correlation is particularly noteworthy for the potential confusion it can cause.
Statistical analysis of a data set often reveals that two variables (properties) of
the population under consideration tend to vary together, as if they were
connected.
(III)
Let students know below statistical measures may be tested by ETS in basic
descriptive statistics. Give Formula to calculate all measures. Measures are
1. Mean
Hints: The mean (or average) is the most popular and well known measure of
central tendency. It can be used with both discrete and continuous data. The
mean is equal to the sum of all the values in the data set divided by the number

of values in the data set. So, if we have n values in a data set and they have

x
2.
3.
4.

5.

6.

7.

values x1, x2, ..., xn, then the sample mean, usually denoted by
(pronounced x
bar)
Median
Hints: The median is the middle score for a set of data that has been arranged in
order of magnitude.
Mode
Hints: The mode is the most frequent score in our data set.
Range
Hints: In the descriptive statistics, the range of a set of data is the difference
between the largest and smallest values. It is the smallest interval which
contains all the data and provides an indication of statistical dispersion
Standard deviation
Hints: This is most important area to be tested in statistics. In statistics and
probability theory, standard deviation (represented by the symbol sigma, )
shows how much variation or "dispersion" exists from the average (mean, or
expected value). A low standard deviation indicates that the data points tend to
be very close to the mean, whereas high standard deviation indicates that the
data points are spread out over a large range of values.
Variance
Hints: In probability theory and statistics, the variance is a measure of how far a
set of numbers is spread out. It is one of several descriptors of a probability
distribution, describing how far the numbers lie from the mean (expected value)
Inter quartile range
Hints: inter quartile range (IQR), also called the mid-spread or middle fifty, is a
measure of statistical dispersion, being equal to the difference between the
upper and lower quartiles IQR = Q3 Q1

Q. D
8. Quartile Deviations

Q3 Q1
2

SPECIAL TIPS FOR STATISTICS:


(i)

Let students know some time ETS tests high difficulty level of questions for

statistics
to handle such problems these special tips are required.
(ii)

|Median-Mean| <= SD

(iii)

Variance is the square of the standard deviation

(iv)

If Range or SD of a list is 0, then the list will contain all identical elements. And
vise versa: if a list contains all identical elements then the range and SD of a list
is 0. If the list contains 1 element: Range is zero and SD is zero.

(v)

SD is always >=0. SD is 0 only when the list contains all identical elements (or
which is same only 1 element)

(vi)

Symmetric about the mean means that the shapes of the distribution on the
right and left side of the curve are mirror-images of each other

(vii)
(viii)

If we add or subtract a constant to each term in a set:


Mean will increase or decrease by the same constant.
SD will not change

(ix)

If we increase or decrease each term in a set by the same percent:

(x)

Mean will increase or decrease by the same percent

(xi)

SD will increase or decrease by the same percent

(xii)

Changing the signs of the element of a set (multiplying by -1) has no effect on
SD.

(xiii)

The SD of any list is not dependent on the average, but on the deviation of the
numbers from the average. So just by knowing that two lists having different
averages doesn't say anything about their standard deviation - different
averages can have the same SD.

5. DATA INTERPRETATION (For Integrated Reasoning Section)


(I)

Define Data (Collection of information about college, university, city, country,


company or any other aspect which really exist this universe known as data)

(II)

Define Data Interpretation (Data Interpretation questions are grouped together


and refer to the same table, graph or other data presentation. These questions
ask you to interpret or analyze the given data. The types of questions may be
Multiple-choice (both types) or Numeric Entry.)

(III)

Mainly following kinds of graphs are tested by ETS in GRE


1. Interpretation of data in tables and graphs (explain with your choice of
examples)
2. line graph (explain with your choice of examples)
3. Bar graphs (explain with your choice of examples)

4. Circle graphs (explain with your choice of examples)


5. Box plots (explain with your choice of examples)
6. Scatter plots (explain with your choice of examples)
(IV)

(V)

Give some tips to answer DI problems


1. Scan the data presentation briefly to see what it is about, but do not spend
time studying all of the information in detail. Focus on those aspects of the
data that are necessary to answer the questions. Pay attention to the axes and
scales of graphs; to the units of measurement or orders of magnitude (such as
billions) that are given in the titles, labels and legends; and to any notes that
clarify the data.
2. Bar graphs and circle graphs, as well as other graphical displays of data, are
drawn to scale, so you can read or estimate data visually from such graphs.
For example, you can use the relative sizes of bars or sectors to compare the
quantities that they represent, but be aware of broken scales and of bars that
do not start at 0.
3. The questions are to be answered only on the basis of the data presented,
everyday facts (such as the number of days in a year) and your knowledge of
mathematics. Do not make use of specialized information you may recall from
other sources about the particular context on which the questions are based
unless the information can be derived from the data presented.
The most important point to understand for students is to use approximation
techniques to answer DI problems. Give detail idea about approximation
techniques by taking some examples.
Examples: OG 13th Edition
Chapter 5: Q101, 112, 132, 185, 201
Chapter 6: Q20

SESSION-12-CLASS ASSIGNMENT (HAND OUTS-MODULE-4-STATISTICS)


1) There are 1000 people of whom 60% are from Maharashtra. 50% are Graduate. 20% of
Maharashtra is Graduate. Find the number of people who are non Maharashtra and non
graduate?
A) 20
B) 40
C) 60
D) 480
E) 300

2) Of the 84 members who attended the meeting at a school, 35 volunteered to supervise


children during the school picnic and 11 volunteered both to supervise children during the
picnic and to bring refreshment to the picnic. If the number of parents who volunteered to
bring refreshment was 1.5 times the number of parents who neither volunteered to
supervise children nor bring refreshment, how many of the parents volunteered to bring
refreshment?
A) 24
B) 36
C) 54
D) 50
E) 60
3) In town X, 64 percent of the population are employed, and 48 percent of the population
are employed males, what percent of the employed people in town X are female?
A) 15%
B) 35%
C) 20%
D) 25%
E) 10%
4) Thirty percent of the members of a swim club have passed the life saving test. Among the
members who have not passed the test, 12 have taken preparatory course and 30 have
not taken the course. How many members are there in the swim club?
A) 40
B) 50
C) 60
D) 70
E) 80
5) If 75 percent of a class answered the first question on a certain test correctly, 55 percent
answered the second question on the test correctly and 20 percent answered neither of
the questions correctly. What percent answered both correctly?
A) 50%
B) 25%
C) 75%
D) 80%
E) 60%
6) Three hundred students at college Q study a foreign language. Of these, 110 of those
student study French, and 170 study Spanish. If at least 90 students who study a foreign
language at college Q study neither French nor Spanish, then the number of students who
study Spanish but not French could be any number from.
A) 50 to 100
B) 100 to 130
C) 130 to 190
D) 60 to 100
E) 50 to 100
7) A marketing firm determined that, of 200 household surveyed, 80 used neither Brand A
nor Brand B soap, 60 used only Brand A soap, and for every household that used both
Brands of soap, 3 used only Brand B soap. How many of 200 household surveyed used
both Brands of soap?
A) 10
B) 15
C) 25
D) 40
E) 50
8) If m is the average of the first 10 positive multiples of 5 and if M is the median of the first
10 positive multiples of 5, what is the value of M m?
A) 5
B) 0
C) 5

D) 25
E) 27.5
9) If Q is an odd number and the median of Q consecutive integers are 120, what is the
largest of these integers?
A)

Q1
2

B)

Q
2

+ 119

C)

Q
2

+ 120

D)

Q+120
2

E)

Q+120
2

+ 120

10) For the positive numbers, n, n + 1, n +2, n + 4, and n + 8, the mean is how much
greater than the median?
A) 0
B) 1
C) n + 1
D) n + 2
E) n + 3
11)
List L: 3, 7, 24, 26, x
If x < 5, which of the following describes all the values of x such that average of list L is
less than the median of list L?
A) X < - 45
B) X < - 25
C) 45 < X < - 25
D) 25 < X < - 10
E) X < - 5

12) The integers v, w, x, y, and z are such that 0 < x < w < x < y < z. The average of these
integers is 36, and the median of these 5 integers is 28. What is the greatest possible
value of z?
A) 128
B) 130
C) 140
D) 132
E) 120
13)
List i: 3, 6, 8, 19
List ii: x, 3, 6, 8, 19
If the median of the numbers in list one above is equal to the median of the numbers in
list second above, what is the value of x?
A. 3
B. 5
C. 7
D. 8
E. 10
14)
5,6,5,6,7,5,5,n,6
For the numbers listed above, the only mode is 5 and the median is 6. Each of the
following could be the value of n EXCEPT?

A)
B)
C)
D)
E)
15)

6
7
8
9
10
3, k, 2, 8, m, 3

The arithmetic mean of the list of numbers above is 4. If k and m are integers and k

m, what is the median of the list?


A) 5
B) 2
C) 3
D) 7
E) 10
16) The numbers of cars sold at a certain dealership on six of the last seven business days
were 4, 7, 2, 8, 3, and 6, respectively. If the number of cars sold on the seventh business
day was either 2, 4, or 5, for which of three values does the average number of cars sold
per business day for the seven business days equal the median number of cars sold per
day for the seven days?
I.
2
II.
4
III.
5
(A) II only
(B) III only
(C) I and II
(D)II and III
(E) I, II, and III

17) An Olympic diver received the following scores: 6.0, 5.5, 7.0, 6.5, and 5.0. The standard
deviation of these scores is in which of the following ranges?
A) 0 1.9
B) 2 3.9
C) 4 6.9
D) 7 7.9
E) 8 9.9
18) A certain characteristic in a large population that is symmetric about the mean m. if 68
percent of the of the distribution lies within one standard deviation d of the mean, what
percent of the distribution is less than m + d?
A) 16%
B) 32%
C) 48%
D) 84%
E) 92%
19)

A Bell Curve (Normal Distribution) has a mean of 1 and a standard deviation of

.How many integer values are within three standard deviation of mean?
A) 1
B) 2
C) 3
D) 4
E) 5
20)
I. 72, 73, 74, 75, 76

1
8

II. 74, 74, 74, 74, 74


III. 62, 74, 74, 74, 89
The data sets I, II, and III above are ordered from greatest standard deviation to list
standard deviation in which of the following?
A) III, I, II
B) III, II, I
C) II, I, III
D) I, II, III
E) II, III, I

Answer key
1. A
2. B
3. D
4. C
5. A
6. D
7. B
8. B
9. A
10.B
11.B
12.E
13.C
14.A
15.C
16.B
17.A
18.D
19.A

20.

Homework Assignment:
OG 12th Edition

Set theory: Problem solving: 178 Data sufficiency: 50, 89


Statistics: Problem solving: 69, 199, 210

Data sufficiency: 81, 129, 134, 147

EGE Math Workbook: Chapter 5 Data Analysis

Vous aimerez peut-être aussi